The words you are searching are inside this book. To get more targeted content, please make full-text search by clicking here.

Rapid Review of Clinical Medicine for MRCP Part 2 by Sharma, Sanjay Kaushal, Rashmi (z-lib.org)

Discover the best professional documents and content resources in AnyFlip Document Base.
Search
Published by srini vasan, 2020-04-28 03:03:52

Rapid Review of Clinical Medicine for MRCP Part 2 by Sharma, Sanjay Kaushal, Rashmi (z-lib.org)

Rapid Review of Clinical Medicine for MRCP Part 2 by Sharma, Sanjay Kaushal, Rashmi (z-lib.org)

Clinical Cases 99

Answer 88

d. Organize CT-guided aspiration/biopsy of the primary infection. The lag period between primary TB
lower lumbar spine. and spinal TB may be decades. Spinal TB may occur as an
immediate complication of the primary infection and
The presentation of back pain associated with weakness most commonly affects the lower thoracic and lumbar
of the lower limbs is consistent with cord compression. spine. Patients present with back pain, night sweats,
The accompanying fever, night sweats and raised weight loss and signs of cord compression from epidural
inflammatory markers are highly suggestive of infection abscess formation. The differential diagnosis is
of the lower spine or a involvement of bone with osteomyelitis secondary to Staphylococcus aureus
haematological malignancy. The plain lumbar spine film infection, in which blood cultures are positive in over
shows minor destruction of the third lumbar vertebra and 90% of cases.
reduced joint space between the third and fourth lumbar
vertebrae to indicate a discitis. The MRI scan shows A chest X-ray is rarely helpful in the diagnosis since it
destruction of the third and fourth lumbar vertebrae and is only abnormal in 50% of cases. Whereas a Heaf test is
marked discitis (reduced joint space). There is a positive in over 90% of immunocompetent patients with
collection of pus around the anterior and posterior spinal TB, many patients with spinal TB are
aspects of the third and fourth lumbar vertebrae. The immunocompromised and may have a negative test. The
anterior collection is pressing on the spinal cord. The investigation of choice following MRI of the spine is a
most probable diagnosis is tuberculous osteomyelitis of CT-guided aspirate/biopsy of the affected area before
the lumbar spine in a female of her ethnicity. starting treatment. An attempt to make a tissue diagnosis
should be made whenever possible before initiating anti-
Spinal TB usually occurs in immunocompromised TB therapy. Treatment is with conventional anti-TB
adults and is due to reactivation of TB acquired from a treatment for 12–18 months.

Question 89 What is the best method of preserving renal function?
a. Cessation of smoking.
A 23-year-old woman with insulin-dependent diabetes b. Strict glycaemic control.
mellitus had experienced recurrent episodes of cystitis for c. ACE inhibitor therapy.
which she was taking trimethoprim. She smoked 15–20 d. Lifelong prophylactic antibiotics.
cigarettes per day. Her blood pressure measured e. Angiotensin II receptor blocker.
120/80 mmHg. Urinalysis did not reveal any evidence of
micralbuminuria.

Question 90 Hb 12 g/dl
WCC 12 ϫ 109/l
A 35-year-old woman presented with vomiting and Platelets 100 ϫ 109/l
epigastric discomfort after a weekend of binge drinking. MCV 102 fl
On examination she was alert. She had several spider Sodium 132 mmol/l
naevi on the face and chest wall. The heart rate was Potassium 3.3 mmol/l
100 beats/min and regular. The blood pressure was Urea 7 mmol/l
160/96 mmHg. The respiratory rate was 20/min. Both Creatinine 80 ␮mol/l
heart sounds were normal. The abdomen was soft. Bicarbonate 11 mmol/l
Inspection of the fundi was normal. Chloride 98 mmol/l
Bilirubin 18 ␮mol/l
Initial investigations are shown. AST 160 iu/l
Gamma GT 100 iu/l
What is the diagnosis? Plasma osmolality 290 mOsm/l
a. Diabetic ketoacidosis. Blood glucose 5 mmol/l
b. Methanol poisoning. Urinalysis Ketones ++++
c. Euglycaemic ketoacidosis. No crystals
d. Renal tubular acidosis.
e. Lactic acidosis.

100

Answer 89

b. Strict glycaemic control. control of blood pressure and the intraglomerular
pressure, particularly using ACE inhibitors or angiotensin
The patient does not have hypertension or micro- II receptor blockers, can greatly retard the progression of
albuminuria, therefore the best treatment for preventing nephropathy (see Algorithm).
nephropathy is meticulous glycaemic control. Glycaemic
control and blood pressure are the main predictors for the In young type 1 diabetics with microalbuminuria or
development of microalbuminuria (the earliest marker of proteinuria, an ACE inhibitor should be prescribed
diabetic nephropathy) in both type 1 and type 2 diabetes. regardless of the blood pressure. The target blood
Data from the DCCT trial in type 1 diabetes and UKPDS pressure (which may require multiple other anti-
trial in type 2 diabetes have shown that meticulous hypertensive agents in addition to ACE inhibitors) in
glycaemic control alone delays the onset of diabetic patients with microalbuminuria is <120/70 mmHg and
nephropathy in patients who do not have coexistent in patients with proteinuria it is <130/75 mmHg.
hypertension. In patients with diabetes and hypertension
prevention of diabetic nephropathy requires both In patients with type 2 diabetes there is more
meticulous glycaemic control and maintaining a normal compelling evidence from large trials that angiotensin II
blood pressure. Better glycaemic control reduces glomerular receptor blockers prevent development of proteinuria in
basement membrane thickening and microalbuminuria. patients with microalbuminuria. Target blood pressure is
<130/75 mmHg.
Once microalbuminuria has developed there is little
evidence that improving glycaemic control alone delays Smoking has also been identified as a risk factor for the
progression of nephropathy. In these circumstances, development of microalbuminuria. However, there are no
prospective studies showing the benefit of cessation of
smoking on renal function in type 1 diabetics at least.

Prevention and management of established diabetic nephropathy
Diabetes

Microalbuminuria* No microalbuminuria

ACE inhibitor (type 1 diabetes) Yes No
Angiotensin receptor blocker (type 2 diabetes)
Meticulous glycaemic Meticulous glycaemic
Ϯ other antihypersensitive agents control and aggressive control Hblc ≤7%
Target BP <120/70 mmHg
BP control
*In patients with frank proteinuria Target BP
target BP is <130/75 mmHg 130/75 mmHg

Answer 90

c. Euglycaemic ketoacidosis. have normal or low blood glucose, but a high glucose is
also recognized. There is no clear explanation for high
The patient is admitted after an episode of alcohol glucose levels except that some patients may have
bingeing. Apart from the history, her investigations raise concomitant diabetes mellitus.
the suspicion of alcohol abuse (macrocytosis, low
platelets and abnormal liver function tests). Patients who The plasma osmolality may also be increased owing to
abuse alcohol may present with ketoacidosis in the alcohol itself, and the generation of endogenous acids but
absence of insulin deficiency. There are several factors also raises the differential diagnosis of methanol or
that contribute to ketoacidosis. Firstly, alcohol inhibits ethylene glycol poisoning. Therefore it is important to
gluconeogenesis, which offsets glucagon-mediated take a detailed alcohol history and to check the urine for
lipolysis and ketone generation. Secondly, alcohol itself ethylene glycol crystals. Salicylate toxicity may also present
may be metabolized to ketones. Thirdly, patients who in a similar fashion and should be excluded by measuring
abuse alcohol generally have a low calorie intake and are plasma salicylate levels. The treatment is IV dextrose and
effectively starving on a chronic basis. Patients usually saline solutions as well as thiamine replacement. The
dextrose replaces the glucose required to inhibit glucagon
production and the saline replenishes fluid loss.

Clinical Cases 101

Question 91 Which two of the following would be possible
options in stepping up this patient’s asthma
A 27-year-old female artist was seen in the chest clinic management?
with a three-day history of worsening dyspnoea requiring
the use of an inhaled short-acting ␤2-agonist every 3–4 a. Continue current treatment and add regular oral
hours. Over the past few months she had been using the prednisolone.
bronchodilator with increasing frequency. She had also
experienced a non-productive cough particularly at night b. Continue short-acting β2-agonist as required
and after exercise. She was diagnosed with asthma since and substitute high-dose inhaled steroid twice
the age of 14 years and was hospitalized on two daily.
occasions more than 10 years ago for infective
exacerbations of asthma. Her only other medication was c. Start regular short-acting inhaled β2-agonist and
a low-dose inhaled steroid, which she takes twice daily. continue on low-dose inhaled steroid twice
daily.
On examination she was afebrile. She was 1.52 m tall.
The peak expiratory flow rate was 365 l/min, with her d. Start regular short-acting β2-agonist and
normal best at 470 l/min. She could complete sentences substitute high-dose inhaled steroid twice daily.
in one breath. Her respiratory rate was 20/min, pulse
rate 80 beats/min and blood pressure 110/70 mmHg. e. Continue short-acting β2-agonist as required
Auscultation of the lungs revealed mild generalized and continue on low-dose inhaled steroid, add
polyphonic wheeze. long-acting inhaled β2-agonist twice daily.

Blood gases (air): f. Continue short-acting β2-agonist as required,
add sodium cromoglycate and stop low-dose
PaCO2 5 kPa inhaled steroid.
PaO2 12 kPa
pH 7.34 g. Continue short-acting β2-agonist as required,
add modified release theophylline and stop low-
dose inhaled steroid.

h. Continue current treatment and add inhaled
ipratropium bromide twice daily.

i. Continue current treatment and add regular oral
prednisolone and erythromycin 250 mg QDS.

j. Continue current treatment and add high-dose
inhaled steroid twice daily.

Question 92 What is the best treatment for reducing the rate of
nephropathy?
A 52-year-old male with non-insulin-dependent diabetes
mellitus has a blood pressure of 148/94 mmHg. a. Losartan.
Fundoscopy reveals evidence of background diabetic b. Ramipril.
retinopathy. c. Insulin.
d. Metformin.
Investigations are shown. e. Atenolol.

Hb 13.1 g/dl
WCC 5 ϫ 109/l
Platelets 290 ϫ 109/l
Sodium 138 mmol/l
Potassium 4.1 mmol/l
Urea 9 mmol/l
Creatinine 138 μmol/l
Glucose 8 mmol/l
Cholesterol 5.8 mmol/l
Triglycerides 3.2 mmol/l
12-lead ECG Normal
24-hour urine protein 1g

102

Answer 91

b. Continue short-acting ␤2-agonist as required and The patient described had none of the features of
substitute high-dose inhaled steroid twice daily. acute severe or life-threatening asthma and had a PEFR
>75% of predicted. She was on step 2 of the management
e. Continue short-acting ␤2-agonist as required and ladder of chronic asthma. She had symptoms indicating
continue on low-dose inhaled steroid, add long- loss of control of asthma and the need to step up
acting inhaled ␤2-agonist twice daily. management (Table B).

The British Thoracic Society (BTS) published guidelines Patients should start treatment at the step most
for the management of asthma in 1990, which have since appropriate to the initial severity. A rescue course of
been updated (1997), in response to concerns over the prednisolone may be needed at any time and at any step.
increasing prevalence, morbidity, admissions and deaths Until growth is complete any child requiring inhaled
from asthma. Supported by randomized controlled trials, high-dose steroids or oral steroids should be referred to a
steroids have since been established as the mainstay of paediatrician with an interest in asthma. The aim is to
preventative treatment in all but the mildest cases of achieve control and then to reduce treatment.
asthma. This question tests the candidate’s knowledge of
the BTS guidelines of the stepwise approach to the A peak-flow meter should be prescribed and the
management of chronic asthma with the introduction of response to treatment should be reviewed every 3–6
inhaled steroids early in the treatment (Table A). months. In patients with chronic asthma a 3–6 month
period of stability should be shown before slow stepwise
reduction in the treatment is undertaken.

Table A Stepwise management of asthma Table B Symptoms indicating inadequate
control of asthma
Step 1 Occasional use of inhaled short-acting
␤2-agonist as required for symptom relief. • The need for frequent relieving bronchodilator
If needed more than once daily move to therapy
step 2
• Chronic cough and nocturnal symptoms
Step 2 Occasional use of inhaled short-acting • Limitations on activity including exercise
␤2-agonist as required plus low-dose • PEFR <80%
inhaled steroid twice daily • Frequent exacerbations
Alternatively use cromoglycate but start • Circadian variation in PEFR >20%
inhaled steroids if control not achieved. • Adverse effects from medications

Step 3 Occasional use of inhaled short-acting
␤2-agonist as required plus high-dose
inhaled steroid daily or low-dose inhaled
steroid twice daily plus long-acting ␤2-
agonist twice daily (especially if
experiencing side-effects with high-dose
inhaled steroids)
Alternatively use MR theophylline or
cromoglycate but start inhaled steroids if
control not achieved

Step 4 Occasional use of inhaled short-acting
␤2-agonist as required plus high-dose
inhaled steroid daily plus sequential
therapeutic trial of one or more of the
following:
• Long-acting ␤2-agonist twice daily
• MR theophylline
• Inhaled ipratropium bromide or
oxitropium
• Long-acting ␤2-agonist tablets
• Cromoglycate

Step 5 Occasional use of inhaled short-acting
␤2-agonist as required plus high-dose
inhaled steroid daily plus one or more of
the long-acting bronchodilators plus
regular prednisolone tablets in a single daily
dose

Clinical Cases 103

Answer 92

a. Losartan. ACE inhibitors in treating diabetic nephropathy in type 2
diabetes mellitus. However, two important studies have
The patient has moderate proteinuria and abnormal renal shown that in patients with type 2 diabetes, angiotensin
function. The question specifically relates to the receptor blockers are as effective in retarding nephropathy
treatment of diabetic nephropathy in a patient with type as ACE inhibitors are in type 1 diabetes mellitus.
2 diabetes mellitus. While ACE inhibitors have been
clearly shown to attenuate micro-albuminuria in patients For example, in the RENAAL trial, 1513 patients with
without overt nephropathy and retard progression of type 2 diabetes and nephropathy were randomly assigned
nephropathy in those patients with established diabetic to losartan (50 titrating up to 100 mg once daily) or
nephropathy in type 1 diabetes, the management of placebo, both in addition to conventional antihyper-
nephropathy in type 2 diabetes is still evolving. tensive therapy (but not ACE inhibitors). Compared to
placebo, losartan reduced the incidence of a doubling of
It is clear that tight glycaemic control is effective in the plasma creatinine by 25% and end-stage renal disease
retarding nephropathy in both type 1 and type 2 diabetes by 28%; the mean follow-up was 3.4 years. These benefits
mellitus and that control of hypertension is also necessary were not associated with differences in blood pressure
in both conditions. In contrast with type 1 diabetes levels between the groups. Subsequent analysis found
mellitus, there is much less information on the effects of that the most significant risk factor for progressive kidney
disease was the initial degree of proteinuria.

Question 93

A 45-year-old man with HIV syndrome presented with What is the most probable diagnosis?
weakness affecting the left upper and lower limbs. On a. Primary CNS lymphoma.
examination he was confused and had a temperature of b. Tuberculous meningitis.
38°C (100.4°F). He had recently been treated for a c. CMV encephalitis.
Pneumocystis carinii infection. The CD4 count was 150. d. Cerebral toxoplasmosis.
CT scan of the brain with contrast revealed subcortical e. Progressive multifocal leucoencephalopathy.
atrophy and multiple contrast-enhancing ring lesions in
the cortex and subcortical areas.

Question 94

A 60-year-old male was noted to have an irregular pulse. blood pressure was 110/80 mmHg and both heart
A subsequent 12-lead ECG showed atrial fibrillation with sounds were normal.
a ventricular rate of 80 beats/min. The patient was
asymptomatic. There was no past medical history of note. Investigations are shown.
The patient consumed 10 units of alcohol per week. His

Hb 12 g/dl Which additional treatment is required to reduce the
Blood glucose 4.2 mmol/l risk of stroke?
Total cholesterol 5.4 mmol/l
TSH 1.3 mu/l a. Warfarin.
Total thyroxine 170 nmol/l b. Clopidogrel.
Chest X-ray Normal c. Ramipril.
12-lead ECG Atrial fibrillation d. Aspirin.
Normal QRS complexes e. Pravastatin.
Echocardiography Left atrial diameter 40 mm
Normal left ventricle
Normal systolic function
Normal valves

104

Answer 93

d. Cerebral toxoplasmosis. medullary junction or around the basal ganglia. Patients
often have headache, confusion and fever. Focal
The most probable diagnosis is cerebral toxoplasmosis. neurological signs or seizure are common. The other
The CT scan is consistent with the diagnosis; the sub- opportunistic infections rarely result in mass lesions in the
cortical atrophy probably represents HIV encephalopathy. absence of disseminated infection.

The exact cause of CNS involvement in HIV infection Primary CNS lymphoma does display some degree of
is largely influenced by the CD4 count. Patients with contrast enhancement but it is usually nodular or patchy;
CD4 counts >500 have benign and malignant brain however, lesions may be multiple and difficult to
tumours similar to those seen in immunocompetent differentiate from toxoplasmosis. Thallium single photon
patients. Patients with a CD4 count of between 200 and emission computed tomography has been shown to be
500 often have cognitive disorders associated with HIV useful in differentiating cerebral toxoplasmosis from
such as HIV dementia and progressive leucoence- cerebral lymphoma. Primary CNS lymphoma is rarer than
phalopathy, which are not mass lesions. Patients with a cerebral toxoplasmosis in HIV infection. Generally
CD4 count <200 have either opportunistic CNS patients with multiple ring-enhancing lesions on the CT
infections or HIV-related cerebral tumours. scan with mass effect are treated empirically for
toxoplasma infection for two weeks with a combination
Opportunistic infections include toxoplasmosis, TB, of pyrimethamine and sulphadiazine. Failure of a
CMV and cryptococcal meningitis (see Question 277). radiological response (on CT scan) is an indication for
Toxoplasmosis is by far the commonest cerebral mass brain biopsy to enable an alternative diagnosis such as
lesion seen in HIV patients and is usually associated with lymphoma.
multiple ring-enhancing lesions either in the cortico-

Answer 94 Treatment to prevent systemic
thromboembolism in atrial fibrillation
d. Aspirin.
Factor Risk Treatment
The patient has atrial fibrillation which is associated with
a five-fold increase in stroke secondary to systemic Any of High Warfarin
thromboembolism. The two main pharmacological Age ≥75 years
treatments available for preventing systemic thrombo- Previous stroke or TIA
embolism in AF are antithrombotic agents and oral Uncontrolled
anticoagulant drugs. Aspirin is the only antithrombotic
agent recommended for preventing systemic thromobo- hypertension
embolism in AF. Warfarin is the most commonly used Impaired left ventricular
oral anticoagulant drug used in this particular situation.
systolic function
Studies have shown that in high-risk patients (see Rheumatic mitral
below) warfarin is superior to aspirin in preventing
systemic thromboembolism. However, there is no valve disease
conclusive evidence that warfarin is superior to aspirin in
this regard in low-risk patients. In patients at moderate Any of Moderate Warfarin
risk both aspirin and warfarin appear to be equally Age 65–74 years or
effective in reducing systemic thromboembolism. Controlled
Patients' age above 75 years of age, rheumatic mitral aspirin
valve disease and prior embolic events are the greatest risk hypertension
factors for systemic thromboembolism in AF (Table). Diabetes mellitus
Ischaemic heart disease
The patient in question is at low risk of systemic Thyrotoxicosis
thromboembolism, therefore he should be treated with
aspirin instead of warfarin. All of

An alternative method of determining who will benefit Age <65 years Low Aspirin
from warfarin is to use the CHADS score (right): a score
>2 is an indication of anticoagulation therapy; patients Normal left
with a score of 1–2 could be considered for aspirin or
warfarin; and patients with a score of 0 are deemed at ventricular function
relatively low risk and suitable for aspirin (150–300 mg).
No evidence of

rheumatic heart disease

No hypertension

C = coronary artery disease 1 point
H = hypertension 1 point
A = age >75 1 point
D = diabetes mellitus 1 point
S = previous stroke 2 points

Clinical Cases 105

Question 95 Which of the following treatments is most suitable for
the patient?
An 84-year-old fully independent male was admitted with
very transient episodes of dizziness. The 12-lead ECG a. AAIR pacemaker.
showed first degree AV block and left bundle branch b. VVI pacemaker.
block. A subsequent 24-hour ECG revealed episodic c. DDDR pacemaker.
third degree AV block. d. VVIR pacemaker.
e. None of the above.

Question 96

Hb 10 g/dl A patient with long-standing rheumatoid arthritis is
noted to have splenomegaly. Results of haematological
WCC 3.1 ϫ 109/l investigations are shown.

(neutrophils, 1.5; What is the cause of the low white cell count?
a. Folate deficiency.
lymphocytes, 0.7; b. Drug-induced marrow aplasia.
c. Myelodysplastic syndrome.
eosinophils, 0.9) d. Felty’s syndrome.
e. Hodgkin’s lymphoma.
Platelets 108 ϫ 109/l

MCV 80 fl

Bone marrow: Erythroid hyperplasia,

myeloid hyperplasia but lack of

mature forms

Multiple megakaryocytes

Question 97 97

A 50-year-old woman presented with pain in her right
shoulder and weight loss of 4 kg over the past four
weeks. She had been taking non-steroidal anti-
inflammatory drugs with only mild relief of her pain. She
smoked 15–20 cigarettes per day.

Investigations are shown.

Hb 11.2 g/dl

WCC 8.2 ϫ 109/l

Platelets 500 ϫ 109/l

ESR 72 mm/h

Chest X-ray (97)

What is the diagnosis?
a. Pancoast’s tumour.
b. Multiple myeloma.
c. Osteomyelitis affecting the right shoulder joint.
d. Metastatic bone disease.
e. Avascular necrosis of the head of femur.

106

Answer 95

c. DDDR Pacemaker. absence of permanent cardiac pacing. The aim would be
to maintain physiological rhythm and to pace both the
The patient has third degree AV block. The condition is atrium and the ventricle, i.e. he should have a dual
associated with an annual mortality exceeding 15% in the chamber pacemaker (Table). He is fully independent and
should have a rate response facility (R).

Choice of pacemaker

Condition Pacemaker of choice
Pure sustained AF VVI or VVIR
Pure sinus node dysfunction without AF or AAIR (or DDDR)*

evidence of AV block at rapid heart rates DDD or DDDR
(>150/min)
Second or third degree AVB or other
bradyarrhythmias with visible P waves

*Most cardiologists in the UK implant DDD (DDDR) pacemakers in patients with sick sinus syndrome as many
patients go on to develop atrioventricular block. First letter, chamber(s) paced; second letter, chamber(s)
sensed; third letter, mode of sensing (I = inhibition and D = triggering and inhibition); fourth letter (R), rate
response facility.

Answer 96

d. Felty’s syndrome. While Hodgkin’s lymphoma may explain the
splenomegaly and eosinophilia (NB: eosinophila is well
In a patient with chronic rheumatoid arthritis, recognized in rheumatoid arthritis, particularly in
neutropaenia and splenomegaly, the most probable patients with skin vasculitis, pulmonary fibrosis and
diagnosis is Felty’s syndrome. However, other conditions subcutaneous nodules), it is less likely than Felty’s
associated with a low white cell count in rheumatoid syndrome, particularly as the disease would have to be
arthritis include drugs such as penicillamine, gold and fairly advanced to produce pancytopenia as in this case.
methotrexate, which are associated with bone marrow Furthermore, at such an advanced state one would
aplasia. Folate deficiency may also cause pancytopenia. expect the bone marrow to have a better yield for the
The bone marrow does not reveal hypoplasia, therefore diagnosis than it normally has in the very early stages of
drug-induced marrow aplasia is unlikely here. The Hodgkin’s lymphoma. Myelodysplasia may cause a
absence of megaloblasts is against the diagnosis of folate hypercellular marrow and a peripheral eosinophilia but
deficiency. splenomegaly is very uncommon.

Answer 97

d. Metastatic bone disease. had a mastectomy, presumably for treatment of
carcinoma. In this case the lucencies have to be assumed
There are small lucencies in the head of the humerus and to be secondary to disseminated breast carcinoma until
a large lucency in the lateral border of the scapula. There proven otherwise. In the exam always ensure both breast
is only one breast shadow present (on the right). The left shadows are present on chest radiographs in female
breast shadow is absent, indicating that the patient has patients. Evidence of a mastectomy may be a major clue
to the answer in a question – as in this case.

Clinical Cases 107

Question 98

A 22-year-old medical student developed sudden
dysphasia and right-sided weakness while on holiday in
Australia. She was afebrile. Apart from the neurological
abnormality described, there were no other abnormal
physical signs.

Investigations are shown.

Hb 15 g/dl What is the most probable diagnosis?
WCC 10 ϫ 109/l a. Carotid dissection.
Platelets 200 ϫ 109/l b. Encephalitis.
ESR 8 mm/h c. Small intracerebral haemorrhage.
Autoantibody screen Normal d. Paradoxical cerebral embolus via patent foramen
12-lead ECG Normal ovale.
2-D echocardiogram Normal e. Cardiac embolus from paroxysmal atrial
Carotid Doppler study Normal fibrillation.

Question 99 99a

A 50-year-old Jamaican male was admitted with discom-
fort in the right shoulder. On examination he had weak-
ness of abduction of the right upper limb.

Investigations are shown.

Hb 11 g/dl

WCC 11.5 ϫ 109/l

Platelets 200 ϫ 109/l

Sodium 133 mmol/l

Potassium 4.6 mmol/l

Urea 7.4 mmol/l

Calcium 2.76 mmol/l

Albumin 36 g/l

Chest X-ray (99a)

What is the diagnosis?
a. Sarcoidosis.
b. Multiple myeloma.
c. Paget’s disease.
d. Bronchial carcinoma.
e. Retrosternal goitre.

108

Answer 98

d. Paradoxical embolus via patent foramen ovale. more than 4 mm in diameter and associated with
intermittent right-to-left intracardiac shunting are more
A young patient presenting with stroke with a normal likely to be associated with systemic emboli. PFOs may
cardiac examination, 2-D echocardiogram, carotid not be identified on a 2-D echocardiogram. A
Doppler study, autoantibody screen and inflammatory transoesophageal echocardiogram with contrast studies is
markers is highly consistent with the diagnosis of the diagnostic investigation of choice. Patients with
paradoxical embolism via a patent foramen ovale. systemic embolism in association with a PFO with a
demonstrable right-to-left shunt with following Valsalva
A PFO occurs when the primum and secundum septa manoeuvre (provided there is no other obvious cause
fail to fuse completely leaving a small flap-like such as atrial fibrillation or intramural thrombus) may be
communication allowing the possibility of a shunt. She treated by closure using a mechanical devise (Amplatzer
has been on a long flight from Australia and therefore it closure device) via the right superficial femoral vein.
is possible that she had deep-vein thrombi, which have
traversed the PFO and passed into the systemic While cerebrovascular accidents are uncommon in
circulation. PFOs are relatively common and may be young individuals, there are several potential causes due
present in up to 30% of the general population. PFOs to disorders affecting the heart and vasculature supplying
the brain and primary central nervous system (Table).

Causes of stroke in young (≤35 years of age) individuals

Cause Notes

Cerebral haemorrhage Sudden onset severe headache and vomiting

Meningoencephalitis Fever, confusion, meningism, seizures

Neoplasm Headache, focal neurology, seizures

Carotid or vertebral dissection Trauma/pain/unilateral lower cranial nerve palsies

Migraine Usually female; may be precipitated by typical aura

Inflammatory conditions (vasculitides, antiphospholipid Systemic features, raised inflammatory markers,

syndrome, isolated cranial angitis, Takayasu’s disease) positive autoantibody screen

Structural arterial disease Involves carotid artery; associated intracranial

(Fibromuscular dysplasia) aneurysms; renal artery commonly affected

Cerebral vein thrombosis Hyperviscocity, pregnancy, contraceptive pill, ear,

sinus or facial skin infection; headache, focal

neurology, seizures

Thrombophilic states Previous arterial or venous thrombosis

Cardiac causes (severe hypertension, AF, Irregular pulse; murmur

rheumatic heart disease, mechanical valves,

cardiomyopathy, septal defects, PFOs)

Haematological (polycythaemia, thrombocythaemia,

paroxysmal nocturnal haemaglobinuria, sickle cell anaemia)

Mitochondrial disease, e.g. MELAS (see Question 315)

Answer 99 99b

d. Bronchial carcinoma.

There is opacification of the right upper lobe consistent
with a mass lesion. The first rib is eroded on the right
side indicating involvement of bone. The findings are
typical of a Pancoast tumour. The raised calcium may be
secondary to bone metastases or secretion of PTH-
related peptide by the carcinoma. Patients with Pancoast
tumour present with symptoms and signs of pressure on
the brachial plexus, the sympathetic trunk (Horner’s
syndrome) and bone metastases (99b, arrows).

Clinical Cases 109

Question 100

A 67-year-old male with dilated cardiomyopathy is daily, bisoprolol 7.5 mg daily and isosorbide mononitrate
admitted for the third time in four months with SR 60 mg daily.
increasing breathlessness and swollen ankles. He
consumed 2 units of alcohol per week. His medications On examination his heart rate was 70 beats/min and
consisted of furosemide 80 mg daily, ramipril 10 mg regular. The blood pressure measured 80/40 mmHg. The
daily, candesartan 6 mg daily, spironolactone 25 mg JVP was raised. Examination of the precordium revealed a
displaced apex in the sixth intercostal space and a forceful
Hb 11 g/dl right ventricular impulse. Auscultation of the heart revealed
a systolic murmur in the mitral area and a third heart
WCC 7 ϫ 109/l sound. On auscultation of the lungs there was evidence of
bilateral pleural effusions. Investigations are shown.
Platelets 200 ϫ 109/l
What is the most effective management step to
MCV 90 fl improve his symptoms?

Sodium 129 mmol/l a. Add digoxin.
b. Consider mitral valve repair.
Potassium 5.9 mmol/l c. Increase dose of bisoprolol to 10 mg.
d. Implant biventricular pacemaker.
Urea 17 mmol/l e. Increase dose of diuretics.

ECG Sinus rhythm; LBBB

Chest X-ray Enlarged heart; bilateral

pleural effusions

Echocardiography:

Dilated left and right ventricles.

Ejection fraction 20%.

Severe functional mitral regurgitation

Question 101

A 21-year-old woman presented with a painless swelling examination she had a 2 cm painless mass thyroid lump
in her neck. There was no history of a recent viral illness. associated with cervical lymphadenopathy.
The patient did not have symptoms of hyper- or
hypothyroidism. She was not taking any medications. On Investigations are shown.

Hb 14 g/dl What is the most useful investigation in elucidating
ESR 5 mm/h the diagnosis?
TSH 2 iu/l
Chest X-ray Normal a. Thyroid ultrasound.
b. Radio-iodine thyroid scan.
c. Fine-needle aspiration of the thyroid gland.
d. Thyroid myoperoxidase antibodies.
e. Total plasma thyroxine concentration.

Question 102

Hb 6.1 g/dl A 70-year-old woman was investigated for lethargy. On
WCC 3.0 ϫ 109/l examination she appeared slightly pale but all other
Platelets 115 ϫ 109/l aspects of physical examination were normal.
MCV 116 fl
MCHC 37 pg Investigations are shown.
Ferritin 710 ␮mol/l
Bilirubin 35 ␮mol/l What is the diagnosis?
AST 40 iu/l a. Hodgkin's lymphoma and haemolytic anaemia.
LDH 1500 iu/l b. Pernicious anaemia.
Alkaline phosphatase 100 iu/l c. Aplastic anaemia.
Albumin 37 g/l d. Paroxysmal nocturnal haemoglobinuria.
e. Autoimmune haemolytic anaemia.

110

Answer 100

d. Implant biventricular pacemaker. Patients with cardiomyopathy and interventricular
conduction defects are generally more symptomatic than
The patient has severe symptoms despite adequate doses those with normal conduction, owing to the resulting
of an ACE inhibitor, beta-blocker and spironolactone. desynchronization of ventricular contraction. Pacing both
His low systolic blood pressure does not allow additional ventricles (RV is paced conventionally; LV is paced via the
pharmacological therapy such as an angiotensin II coronary sinus) helps resynchronize ventricular contraction
receptor blocker. He has poor left ventricular function and improve functional capacity. Indications for
and an interventricular conduction defect suggesting that biventricular pacing are listed in Table A. Biventricular
he may benefit symptomatically from biventricular pacing pacing is associated with significant improvement in quality
(Table A). The mitral regurgitation is a consequence of of life, functional capacity and reduction in hospitalization
the left ventricular dilatation rather than the cause of it. owing to heart failure (Table B). There are preliminary
An internal cardiac defibrillator prevents sudden death in reports that suggest that overall mortality from heart failure
patients at risk of malignant ventricular arrhythmias but is also improved with biventricular pacing.
does not improve functional capacity.
Table B Clinical effects of biventricular pacing
Table A Indications for biventricular pacing
Improves Reduces
• Severe NYHA class III–IV failure despite Quality of life Total and heart failure
optimal drug therapy related hospitalizations
Exercise capacity Heart rate
• QRS >130 msec NYHA functional class
• Left ventricular end diastolic diameter >55 mm LV ejection fraction
• Left ventricular ejection fraction <35% LV dimensions

Answer 101

c. Fine-needle aspiration of the thyroid gland. presence of a hot nodule. Since these are never malignant
a fine-needle aspiration is not warranted.
Thyroid lumps may represent simple or multinodular
goitres, toxic nodules, neoplastic lesions, inflammatory Unfortunately, fine-needle aspiration cannot dis-
masses or thyroid cysts. Thyroid carcinomas account for tinguish between follicular and Hürthle cell adenomas
5–6% of all lumps in the thyroid. Thyroid cancer is most and carcinomas. In these cases the biopsy is termed
common in patients aged under 30 years or over 60 intermediate rather than obviously benign or malignant.
years. Prior radiation to the head and neck is a recog- In such cases a radio-iodine uptake scan is performed and
nized risk factor for thyroid carcinoma. Patients with a if a cold nodule is identified then it is surgically excised.
family history of thyroid carcinoma are at a higher risk of
thyroid malignancy than the general population. In patients suspected to have Hashimoto’s thyroiditis
clinically, or on the basis of high thyroid peroxidase
A painless swelling in the neck associated with cervical antibodies, a thyroid uptake scan should be performed
lymphadenopathy is highly suggestive of papillary before fine-needle aspiration as this may reveal func-
carcinoma of the thyroid gland. Papillary carcinoma of tioning thyroid tissue in the entire lobe and obviate the
the thyroid invades local structures quickly and usually need for biopsy. This is particularly important as patients
presents early as a result. The diagnosis of thyroid with Hashimoto’s thyroiditis have Hürthle’s cells on
carcinoma is usually made by histological sampling of a histology, which may be mistaken for a Hürthle cell
fine-needle thyroid aspirate. Prior thyroid ultrasound is carcinoma.
not necessary. Indeed in patients with a normal TSH,
fine-needle aspiration is the investigation of choice to While ultrasonography would provide more detail
investigate thyroid lumps. In patients with a low TSH a about the morphology of the gland, it is not diagnostic.
radio-iodine uptake scan is performed to confirm the Thyroid scintigraphy is sometimes used by some centres
to ascertain the functional status of the gland, but in this
particular situation it would not provide a diagnosis.

Clinical Cases 111

Answer 102

b. Pernicious anaemia. nucleoside. Erythroid precursors are larger than
reticulocytes. Indeed a MCV >115 fl should always raise
This patient with lethargy has a markedly raised MCV the possibility of abnormal DNA maturation in red cells,
and pancytopenia. The best answer is pernicious anaemia. e.g. B12 deficiency, folate deficiency, drugs interfering
Although paroxysmal nocturnal haemoglobinuria can with DNA synthesis.
also be associated with pancytopenia and raised MCV,
the actual MCV in PNH is less than in pernicious The raised ferritin is due to iron overload resulting
anaemia. The raised MCV in PNH is due to reticulocytes from reduced red cell synthesis. The raised LDH and
(newly formed red cell) arising from rapid red cell AST are due to haemolysis. Immature red cell precursors
turnover secondary to haemolysis, whereas in pernicious undergo extravascular haemolysis. Aplastic anaemia
anaemia the raised MCV is due to large immature red would explain the pancytopenia and raised ferritin but
cells (erythroid precursors) secondary to immature not the raised MCV or haemolysis. Hodgkin’s lymphoma
nuclear development. Remember hydroxycobalamin is with haemolysis could explain all of the data given in the
required for thymidine synthesis, an important DNA question but the MCV favours pernicious anaemia. (See
Answer 394.)

Question 103 Hb 15 g/dl
WCC 8 ϫ 109/l
A 64-year-old man presented immediately after Platelets 300 ϫ 109/l
recovering from a 20-minute episode of dysphasia and ESR 20 mm/h
weakness of the right side of the face and arm. He had Sodium 138 mmol/l
experienced two episodes of transient loss of vision in the Potassium 4.1 mmol/l
left eye in the past three weeks. There was a past medical Urea 6 mmol/l
history of a myocardial infarction two years ago. His Creatinine 100 mmol/l
medication comprised 75 mg of aspirin daily. He did not Glucose 4.7 mmol/l
smoke. Total cholesterol 6 mmol/l
CT scan brain Normal
On examination there was no evidence of residual Carotid Doppler Stenosis (40%) in left
neurological deficit. Fundoscopy was normal. The pulse internal carotid artery.
rate was 80 beats/min and regular in nature. His blood Stenosis (80%) in right
pressure measured 138/86 mmHg in both arms. Both external carotid artery
heart sounds were normal. Auscultation over the carotid
arteries revealed a bruit over the left carotid artery.

Investigations are shown.

Which three treatments have been shown to reduce
the risk of recurrent stroke in this type of patient?

a. Add clopidogrel to current therapy.
b. Add dipyridamole to current therapy.
c. Substitute dipyridamole for current therapy.
d. Increase dose of aspirin to at least 150 mg.
e. Perform left carotid endarterectomy.
f. Perform right carotid endarterectomy.
g. Start ACE inhibitor after three days.
h. Anticoagulate with warfarin.
i. Start therapy with a statin drug.

112

Answer 103 Management of first presentation of
TIA or stroke
b. Add dipyridamole to current therapy.
g. Start ACE inhibitor after three days. Transient ischaemic attack or stroke
i. Start therapy with a statin drug.
CT scan or MRI scan brain
The patient presents with a transient ischaemic attack
which is defined as a neurological deficit lasting <24 Embolic cerebral event
hours that is attributed to focal cerebral or retinal
ischaemia. The causes of TIAs are the same as those of Aspirin 300 mg first dose
stroke and include cardiac intramural thrombi, carotid followed by 75 mg*
artery disease and disease of the intracranial arteries. Statin if cholesterol >5 mmol/l
Therefore strategies for the prevention of recurrent TIAs Treat hypertension after 72 hours
are similar to those for stroke. Aim for BP ≤130/85mmHg
Advice regarding smoking
Aspirin is the drug of choice in patients presenting cessation, weight reduction and
with TIA or stroke unless there are obvious moderation of alcohol intake
contraindications. Aspirin reduces the long-term risk of
stroke and cardiovascular events after a TIA or stroke, Carotid Doppler study
with an overall relative risk reduction of 22%. The dose
of aspirin used ranges from 75 mg to 1300 mg, but small Internal carotid artery stenosis ≥
doses are as effective as large doses and are associated 70% in appropriate carotid artery
with a lower rate of gastrointestinal effects. The dose
most commonly prescribed in the UK is 75 mg daily. Yes No
The thienopyridines clopidogrel and ticlopidine are not
superior to aspirin in secondary prevention. There are no Consider carotid Consider
data on these drugs in primary prevention. endarterectomy medical therapy

Before commencing therapy, all patients should have *In patients with AF anticoagulate immediately
imaging of the brain (usually CT scan) to rule out with warfarin in the case TIA. In patients with
unsuspected cerebral pathology as a cause for a TIA, such embolic stroke treat with aspirin for two weeks
as a brain tumour or a subdural haematoma, where and then switch to warfarin.
therapy with aspirin is contraindicated (algorithm).

Imaging of the carotid arteries using carotid Doppler
studies is performed after imaging of the brain to exclude
a significant internal carotid artery stenosis. Carotid
endarterectomy is of proven benefit in patients with an
internal carotid artery stenosis of ≥70% who have had a
non-disabling stroke or a TIA attributable to the
stenosis. The optimal timing of surgery is currently
unknown but greatest benefit appears to be derived if
surgery is performed within the first two weeks of a non-
disabling stroke, provided there are no contraindications
to surgery.

Anticoagulation has not been evaluated specifically in
patients with TIA but has been extensively tested after
ischaemic stroke. In patients with atrial fibrillation, long-
term oral anticoagulation reduces the risk of recurrent
stroke. In the absence of AF, oral anticoagulation with
warfarin is not superior to aspirin in reducing the risk of
recurrent ischaemic stroke.

Clinical Cases 113

All patients presenting with TIA or stroke require 5 mmol/l in patients with TIA or stroke. In asympto-
meticulous control of blood pressure. Sudden lowering matic patients treatment of hyper-cholesterolaemia is
of the blood pressure may precipitate stroke in patients dependent upon age, and other risk factors for stroke.
with TIAs due to carotid artery stenoses or potentiate
neurological disability in patients with stroke. It is It is important not to underestimate the importance of
generally recommended that blood pressure should be lifestyle modification, and advice regarding smoking
reduced cautiously 72 hours after the neurological event. cessation, exercise, moderation of alcohol consumption,
The aim is to keep blood pressure below and weight control is mandatory.
130/80 mmHg. All antihypertensive agents protect
against stroke; however, the current literature suggests Some patients continue to have TIAs or further
that ACE inhibitors may reduce the risk of further strokes despite the use of aspirin. In such patients the
cerebrovascular events even in patients who were addition of extended-release dipyridamole has been
normotensive at presentation. shown to reduce the risk of further cerebrovascular
embolic events. (European Stroke Prevention Study 2.) A
Lipid-lowering therapy with the statin class of drugs transoesophageal echocardiogram should also be
has been effective in primary and secondary prevention of performed to exclude a patent foramen ovale as a cause of
stroke. The aim is to treat cholesterol levels above paradoxical emboli.

Question 104

A 36-year-old woman presented with a four-week history children, aged 7 and 3 years. She smoked 15 cigarettes
of intermittent upper abdominal pain and vomiting. Her per day and consumed 1–2 units of alcohol per week.
bowel movements were unaffected and were normal.
There was no blood or mucus in the stool. Her appetite On examination, she appeared unwell. Her tongue
was reduced. She had lost 2 kg in weight. She had a past was dry and there was loss of skin turgor. She had mild
medical history of ulcerative colitis which was diagnosed lower limb pitting oedema. The heart rate was 98
at the age of 14 years, and was stable on sulphasalazine. beats/min and blood pressure was 100/65 mmHg. The
In the past she had received several courses of steroids for temperature was 36.8°C (98.2°F). The abdomen was
acute exacerbation of her colitis. Her only other drug slightly distended, and there was generalized tenderness.
history was that she was currently on the contraceptive The liver was palpable 4 cm below the costal margin, and
pill. She had been married for 10 years and had two was tender. There was no evidence of a palpable spleen or
any other palpable masses in the abdomen. Percussion of
Hb 13 g/dl the abdomen revealed shifting dullness. Rectal
WCC 8 ϫ 109/l examination was normal. Examination of the respiratory
Platelets 190 ϫ 109/l and cardiovascular system was normal.
Sodium 131 mmol/l
Potassium 3.1 mmol/l Investigations are shown.
Urea 7.2 mmol/l
Creatinine 100 ␮mol/l 1. What two investigations would you request to
Bilirubin 20 μmol/l ascertain the cause of the presentation?
AST 24 iu/l
ALT 21 iu/l 2. What is the diagnosis?
Alkaline phosphatase 150 iu/l a. Primary sclerosing cholangitis.
Albumin 29 g/l b. Carcinoma of the colon complicated by
Chest X-ray Normal heart size hepatic metastases.
Clear lung fields c. Hepatic vein thrombosis.
d. Cirrhosis of the liver.
e. Chronic active hepatitis.

114

Answer 104

1. i. Liver ultrasound and Doppler studies. vein thrombosis may also occur in ulcerative colitis, the
ii. Liver biopsy via internal jugular vein. abnormal LFT and hepatomegaly would be against the
diagnosis. The differential diagnosis comprises carcinoma
2. c. Hepatic vein thrombosis. of the colon with hepatic metastases, CAH, cirrhosis of
the liver and sclerosing cholangitis. Carcinoma of the
The patient has a long history of relatively quiescent colon is ten times more common in a patient with long-
ulcerative colitis and presents with a short history of standing ulcerative colitis than in the general population;
intermittent upper abdominal pain, vomiting, tender however, it is unusual for a patient with relatively well-
hepatomegaly and ascites. There is no history of weight controlled colitis to present with extensive metastatic liver
loss preceding the illness. However, she is a smoker and disease without a prodrome of ill health or bleeding per
on the oral contraceptive pill, both of which are recog- rectum. Primary sclerosing cholangitis is strongly
nized risk factors for venous thrombosis. Ulcerative associated with ulcerative colitis. Some 75% of all cases
colitis itself is associated with an increased risk of venous with primary sclerosing cholangitis occur in patients with
thrombosis owing to increased fibrinogen levels and the ulcerative colitis. The activity of the colitis is inversely
elevated plasma viscosity. The most likely diagnosis in related to the severity of sclerosing cholangitis. It is
this patient is hepatic vein thrombosis. Although portal usually asymptomatic in the early stages, the only
indicator being a raised alkaline phosphatase. Pruritus,
Complications of ulcerative colitis jaundice, abdominal pain and weight loss are features of
advanced cholangitis. Rapid deterioration may occur
Gastrointestinal when sclerosing cholangitis is complicated by a
• Haemorrhage cholangiocarcinoma. If this patient’s presentation were
• Toxic dilatation due to primary sclerosing cholangitis or cholangio-
• Perforation carcinoma complicating sclerosing cholangitis, one would
• Carcinoma of the colon expect a higher bilirubin level and alkaline phosphatase
• Oral and anal ulcers level, particularly if the associated cirrhosis was severe
enough to cause hepatocellular failure, as is suggested by
Hepatobiliary the low albumin and ascites. Chronic active hepatitis is
• Fatty infiltration of the liver also recognized in ulcerative colitis, but the normal
• CAH transaminase level is against the diagnosis.
• Cirrhosis of the liver
• Sclerosing cholangitis The investigation of choice is liver US with Doppler
studies on hepatic venous flow. Liver biopsy and hepatic
Dermatological venography are also useful in making the diagnosis. A
• Erythema nodosum liver isotope scan may demonstrate preservation of the
• Pyoderma gangrenosum Riedel’s lobe.

Ophthalmic All of the complications mentioned above may also
• Episcleritis complicate Crohn’s disease; however, primary sclerosing
• Scleritis cholangitis, cholangiocarcinoma and the predisposition
• Anterior uveitis to venous thromboses is much more common in
ulcerative colitis. Perianal ulcers are much more common
Rheumatological in Crohn’s disease and, in addition, patients with Crohn’s
• Seronegative arthritis of the small joints disease have a higher incidence of gallstones than those
• Ankylosing spondylitis with ulcerative colitis. The complications of ulcerative
colitis are shown (Table). With the exception of
Haematological ankylosing spondylitis and hepatobiliary disease, all the
• Predisposition to venous thromboses complications of ulcerative colitis are relieved by
proctocolectomy.

Clinical Cases 115

Question 105

A 40-year-old woman presented with a five-day history of was 80 beats/min and regular. The blood pressure was
recurrent falls and an unsteady gait. She had a past 130/90 mmHg. Both heart sounds were normal.
history of a stroke causing a right-sided hemiparesis,
which resolved spontaneously after a few days. The What is the most probable diagnosis?
patient underwent intensive investigation following a. Systemic lupus erythematosus with cerebral
presentation with the stroke including carotid Doppler involvement.
studies, transoesophageal echocardiography and CT scan b. Multiple sclerosis.
of the brain, which were normal. c. Multiple paradoxical emboli.
d. Friedreich’s ataxia.
On examination she had a broad-based gait. There was e. Multiple cerebral metastases.
evidence of dysdiadochokinesia in both upper limbs and
abnormal heel–shin testing. The lower limb reflexes were
brisk and the plantar response was extensor. The heart rate

Questions 106 and 107

A 30-year-old male presented with weight loss and years ago, there was no past history of note. On
palpitation for six months. He had lost approximately examination he had a resting tachycardia and evidence of
4 kg. The patient had been well previously. He was not bilateral gynaecomastia. There were no other abnormalities.
taking any medication. There was no family history of note.
Apart from an orchidopexy affecting the right testicle ten Investigations are shown.

Hb 14 g/dl Question 106
WCC 6 ϫ 109/l
Platelets 180 ϫ 109/l What is the diagnosis?
Sodium 135 mmol/l a. Leydig cell tumour.
Potassium 4.3 mmol/l b. Choriocarcinoma.
Urea 5 mmol/l c. Teratoma.
Albumin 38 g/l d. Seminoma.
Alkaline phosphatase 190 iu/l (NR 25–115 iu/l) e. Thyroid carcinoma.
Alpha-fetoprotein 8 ku/l (NR <10 ku/l)
Human chorionic 1250 iu/l Question 107

gonadotrophin <0.01 mu/l What investigation is required to help make the
TSH 300 nmol/l diagnosis?
Thyroxine
(NR 60–160 nmol/l) a. Ultrasound testes.
Testosterone 60 nmol/l b. CXR.
c. CT thorax.
Oestradiol (NR 10–35 nmol/l) d. FSH level.
1300 pmol/l e. FNA thyroid.

(NR 500–1100 pmol/l)

Question 108 What is the neurological diagnosis?
a. L5 radiculopathy.
A 42-year-old man required ventilation for a prolonged b. L4 radiculopathy.
period during an episode of septicaemia. Following this c. Common peroneal nerve palsy.
he developed difficulty with walking and required the aid d. Femoral nerve palsy.
of an assistant to mobilize. On neurological examination e. Guillain–Barré syndrome.
there was weakness on dorsiflexion of the toes, as well as
ankle eversion. The patient also had reduced sensation,
affecting the anterior lateral aspects below the knee and
the dorsum of the foot.

116

Answer 105

b. Multiple sclerosis. The normal CT scan of the brain and carotid Doppler
studies are against the diagnosis of multiple cerebral
The patient has had a previous right hemiparesis and now metastases and infarctions. A normal transoesophageal
presents with ataxia. Neurological examination reveals echocardiogram is against the diagnosis of paradoxical
findings consistent with a cerebellar syndrome and bilateral emboli. Whereas SLE may manifest with multiple
pyramidal tract involvement. The differential diagnosis of neurological manifestations, the patient does not have
cerebellar and pyramidal tract involvement includes any other manifestations of a multi-system disorder. With
multiple sclerosis, subacute combined degeneration of the specific reference to SLE there is no history of arthralgia,
spinal cord, Friedreich’s ataxia, multiple cerebral metastases myalgia or a rash. Whilst Friedreich’s ataxia may be
and multiple cerebral infarcts. associated with pyramidal tract and cerebellar involve-
ment, the condition usually presents at a much younger
The most probable diagnosis is multiple sclerosis, a age and is associated with progressive neurological
demyelinating disorder characterized by involvement of deterioration. The condition also affects the dorsal root
the optic tracts, pyramidal tracts, cerebellar peduncle and ganglia, which causes absent peripheral reflexes.
the posterior columns of the spinal cord.

Answers 106 and 107

Answer 106 Testicular tumours can be divided into germ-cell
b. Choriocarcinoma. tumours and sex-cord stromal cell tumours. Germ-cell
tumours include seminomas, choriocarcinomas,
Answer 107 teratomas, embryonal cell cancer, and yolk sac tumours.
a. Ultrasound testes. Sex-cord tumours comprise Leydig cell tumours and
tumours affecting the cells of Sertoli. 95% of all testicular
The patient had weight loss, palpitation and suppressed tumours are germ-cell tumours. 50% of all germ-cell
TSH, suggesting hyperthyroidism. Furthermore he has tumours are seminomas. Seminomas are most common
gynaecomastia, raised testosterone and oestrogen levels in the fourth decade. Tumour markers are usually
as well as a high human chorionic gonadotrophin level. normal. Testicular teratomas are relatively rare. They may
This combination is highly suggestive of a chorionic be found in prepubertal males and in adults. Tumour
carcinoma; chorionic carcinomas are characterized by markers are usually negative. Choriocarcinomas are
high HCG levels. The alpha subunit in HCG is similar to associated with high HCG levels.
that found in TSH, FSH and LH, therefore patients with
choriocarcinoma may be hyperthyroid and have raised Leydig cells are the commonest sex-cord stromal
testosterone and oestrogen. tumours that have the ability to produce both oestrogen
and testosterone.

Answer 108

c. Common peroneal nerve palsy. damaged owing to acute or chronic compression.
Common peroneal nerve palsy occurs following
Weakness of dorsiflexion and eversion of the foot and compression due to plaster casts below the knee,
reduced sensation over the antero-lateral aspects of the excessive crossing of the legs, or prolonged pressure on
lower leg and dorsum of the foot are typical features of the fibula during general anaesthesia or unconsciousness.
common peroneal nerve palsy. The common peroneal The nerve may be also be damaged during trauma or
nerve crosses over the fibula, where it may become affected by diabetic neuropathy.

Clinical Cases 117

Question 109

A 53-year-old male was investigated for recurrent attacks in the past 18 months. Just three months
episodes of sinusitis and earache that did not respond to previously he experienced two episodes of epistaxis that
conventional antibiotics. He had experienced several were managed at home. He was generally unwell and
tired easily. He had taken six weeks of sick leave from
Hb 11 g/dl work for malaise and fever in the past six months.
WCC 15 ϫ 109/l According to his wife he seemed to be losing weight. On
(neutrophils 12 ϫ 109/l, further questioning the patient complained of a dry non-
Platelets lymphocytes 2 ϫ 109/l, productive cough usually at the same time that he had
MCV eosinophils 0.35 ϫ 109/l) either sinusitis or earache. He denied haemoptysis or
ESR 490 ϫ 109/l night sweats. He worked as a postman. He did not
CRP 80 fl smoke and consumed alcohol on an infrequent basis.
Sodium 60 mm/h Apart from a holiday in Kenya three years ago, he had
Potassium 90 g/l not left the country for ten years. There was a family
Urea 138 mmol/l history of adult onset asthma. His father had died from
Creatinine 4.1 mmol/l bronchial carcinoma.
Bilirubin 7 mmol/l
AST 110 ␮mol/l On examination, he was thin. There was no clubbing
ALT 11 ␮mol/l or lymphadenopathy. His heart rate was 98 beats/min.
Alkaline phosphatase 23 iu/l The blood pressure measured 148/96 mmHg. The JVP
Albumin 25 iu/l was not raised. Cardiovascular examination was normal.
Immunology: 156 iu/l (NR <115 iu/l) Examination of the upper respiratory tract revealed
31 g/l crusting of the inner aspect to the nostrils. The throat
pANCA was normal. Auscultation of the lung fields was normal,
(myeloperoxidase) Positive as was examination of the abdomen. On neurological
examination, Rinne’s test was positive in the right ear.
cANCA Negative There was no skin rash.
ANF Negative
C3 0.9 g/l (NR 0.55–1.2 g/l) Investigations are shown.
C4 0.3 g/l (NR 0.2–0.5 g/l)
Chest X-ray Right middle lobe What is the most likely diagnosis?
a. Classic polyarteritis nodosa.
Urinalysis consolidation b. Churg–Strauss syndrome.
24-hour protein 0.8g c. Microscopic polyangiitis.
Blood ++ d. Mixed essential cryoglobulinaemic vasculitis.
Casts absent e. Wegener’s granulomatosis.

Question 110 Which one of the following investigations would you
perform next to help identify the cause of his illness?
A 40-year-old homosexual male presented to the
Accident and Emergency Department with a four-day a. Blood film.
history of sore throat and fever. On examination he had b. Monospot test.
cervical and axillary lymphadenopathy and widespread c. HIV viral RNA load.
maculopapular rash. An HIV antibody test was negative. d. CD4 count.
e. CT scan of the thorax and abdomen.

118

Answer 109

c. Microscopic polyangiitis. cANCA (antigen is proteinase 3) whereas microscopic
polyangiitis is characterized by presence of pANCA
The patient presents with specific symptoms of upper and (antigen is myeloperoxidase). The presence of pANCA in
lower respiratory tract involvement. Additionally, he has this case favours the diagnosis of microscopic
malaise, arthralgia, weight loss and raised inflammatory polyangiitis.
markers suggestive of a systemic illness. The presence of
blood and protein in the urine indicates renal involve- Churg–Strauss syndrome is a small-vessel vasculitis
ment. A normal creatinine concentration does not affecting the upper and lower respiratory tract as well as
preclude renal involvement. The differential diagnosis is the skin and the peripheral nervous system. Nasal polyps,
that of a multi-system disease capable of involving the allergic rhinitis and adult-onset asthma usually predate
upper and lower respiratory tract as well as the kidneys. the vasculitis by many years. Renal involvement is usually
The presence of a positive ANCA test is suggestive of a mild but can be severe. A mild eosinophilia (usually <2 ϫ
small-vessel vasculitis. Possibilities include Wegener’s 109/l) is a recognized feature. Patients have a positive
granulomatosis, microscopic polyangiitis and pANCA directed specifically against myeloperoxidase.
Churg–Strauss syndrome. (See Questions 4, 396.)

Wegener’s granulomatosis and microscopic Classic PAN is vasculitis that can affect medium and
polyangiitis are both small-vessel vasculitides associated small vessels. It is characterized by micro-aneurysms,
with ANCA. Both are characterized by upper and lower tissue infarction, haemorrhage and organ dysfunction.
tract involvement (sinusitis, epistaxis, otitis media, Fever, malaise, gastrointestinal, cardiac, CNS and soft
haemoptysis) and renal involvement. Other features tissue involvement is usual. The diagnosis is best made by
common to both include episcleritis, peripheral/central arteriography. Renal biopsy is usually not diagnostic
nervous system involvement and effects on the unless a medium-sized vessel is included in the biopsy
gastrointestinal tract. While manifestations of the upper specimen. The glomeruli are normal. ANCA is charac-
and lower respiratory tract involvement may be identical teristically absent. (See Question 174.)
in both of these small-vessel vasculitides, the presence of
granulomata in small vessels in biopsy specimens is Mixed essential cryoglobulinaemia hepatitis C virus is
specific for Wegener’s granulomatosis. The ANCA implicated in 80% of cases. Clinical features include
subtype may also aid differentiation between Wegener’s pupura, arthralgia, membranoproliferative glom-
granulomatosis and microscopic polyangiitis. Wegener’s erulonephritis, hepatosplenomegaly and neurological
granulomatosis is characterized by the presence of manifestations. Rheumatoid factor levels are often high.
Complement levels are low. ANCA is usually absent.
(See Question 15.)

Answer 110

c. HIV viral RNA load. the diagnosis can be confirmed by measuring HIV viral
RNA load. HIV viral load is not routinely used to
The presentation is consistent with HIV seroconversion. diagnose HIV infection because it is labour intensive, but
Most patients can be diagnosed by conventional ELISA it is useful in diagnosing acute HIV infection, helping to
tests, which identify the presence of HIV antibodies by resolve indeterminate HIV antibody results and in
the time the patient has features of HIV seroconversion. newborn babies where there is passive transfer of
However, if HIV antibody testing proves negative, then antibody from the mother to the baby. A low CD4 count
alone is not specific for HIV infection.

Clinical Cases 119

Question 111 With what are the lung function tests most
consistent?
A 68-year-old male complained of dyspnoea on minimal
exertion. Respiratory function tests are shown. a. Extrinsic allergic alveolitis.
b. Pulmonary fibrosis.
FEV1 2.1 l (predicted 2–3 l) c. Asthma.
FVC 2.5 l (predicted 2.8–4.4 l) d. Ankylosing spondylitis.
TLC 4.2 l (predicted 5–7 l) e. Pulmonary hypertension.
TLCO 90% predicted value

Question 112

A 66-year-old female patient presented with transient 40 years. Physical examination revealed a right-sided
bilateral loss of vision lasting a few seconds. A few days hemiparesis but no other abnormality. The heart rate was
later she developed a right-sided hemiparesis. There was 80 beats/min and regular. The blood pressure measured
no history of head injury or headaches. She had a four- 160/100 mmHg.
year history of hypertension. The patient had smoked for
The following test results were obtained:

12-lead ECG Normal What is the investigation of choice to ascertain the
Normal cause of her presentation?
2-D echocardiogram
with colour flow Left middle cerebral a. Carotid Doppler studies.
artery infarction b. Transoesophageal echocardiograph.
CT scan brain c. Carotid angiography.
d. MRI scan brain.
e. Thrombophilia screen.

Question 113 Hb 13 g/dl

A 79-year-old man was admitted with sudden onset of WCC 13 ϫ 109/l
chest pain and breathlessness. On examination he was
cyanosed. He had had a total hip replacement ten days Platelets 250 ϫ 109/l
previously. The patient smoked ten cigarettes per day and
had hypertension for which he took nifedipine. On CRP 28 g/l
examination the heart sounds were normal and ausculta-
tion of the lungs revealed a clear chest. ECG Sinus tachycardia and LBBB

Investigations are shown. Arterial blood gases (air):

pH 7.49

PaCO2 3.1 kPa
PaO2 8.8 kPa
HCO3 28 mmol/l

What is the most probable diagnosis?
a. Pulmonary embolism.
b. Acute myocardial infarction.
c. Fat embolism syndrome.
d. Pneumonia.
e. Pulmonary oedema secondary to hypertensive
heart disease.

120

Answer 111

d. Ankylosing spondylitis. neuromuscular defect affecting the respiratory muscles or
massive pleural fibrosis. From the options given,
The patient has a restrictive lung defect (FEV1/FVC ankylosing spondylitis is the best answer.
ratio approx. 80%), a low total lung capacity and a
relatively normal transfer factor. These findings are Extrinsic allergic alveolitis and pulmonary fibrosis are
consistent with either a thoracic cage deformity, a associated with a low transfer factor. Asthma causes an
obstructive respiratory defect and pure pulmonary
hypertension does not usually affect the lung volumes.

Answer 112

a. Carotid Doppler studies. The normal transthoracic echocardiogram certainly
excludes calcific thromboemboli from degenerative aortic
The patient has evidence of multiple transient ischaemic valve disease, but a patent foramen ovale cannot be
episodes followed by a right-sided hemiparesis. CT scan excluded with certainty. However, in patients aged over
of the brain reveals a left middle cerebral artery 40, carotid artery disease is a much commoner cause of
infarction, which is consistent with the final presentation. stroke than a patent foramen ovale.
There is no evidence of any other abnormality in the
brain. Possible causes of multiple cerebral emboli in this The initial investigation of choice is a carotid Doppler
particular case include thrombi from the carotid arteries, study (see Answer 166). In the absence of significant
calcific plaques from a degenerative aortic valve, and carotid artery disease, a transoesphageal echocardiogram
paradoxical emboli from a patent foramen ovale. In the should be performed to exclude a patent foramen ovale.
absence of atrial fibrillation, the commonest cause of A thrombophilia screen is indicated in young patients
thromboembolism to the brain is atheromatous carotid with stroke and in those with a previous history of
artery disease, particularly in patients with hypertension. arterial and venous thrombosis or a family history of
arterial or venous thrombosis.

Answer 113

a. Pulmonary embolism. myocardial infarction, the blood gases cannot be
explained by the absence of pulmonary oedema, yet his
The presentation with sudden chest pain, dyspnoea, chest is clear on auscultation.
hypoxia and hypocarbia ten days after a hip replacement
is highly suggestive of pulmonary embolism. The ECG Fat embolism syndrome is a condition that may occur
reveals LBBB, which may be due to silent coronary after bony fractures (more common with closed than
disease or long-standing hypertension. The ECG in open fractures). It is characterized by hypoxaemia,
massive pulmonary embolism classically reveals right petechial rash and neurological abnormalities usually
ventricular strain pattern or RBBB. Although it is within 72 hours (but rarely before 12 hours) after the
possible that the presentation may represent acute procedure. The patient above does not fulfil the diag-
nosis of fat embolism syndrome.

Clinical Cases 121

Questions 114 and 115 Investigations are shown.

A 56-year-old woman was admitted for a left hip Hb 10 g/dl
replacement. She had mechanical mitral valve prosthesis. WCC 12 ϫ 109/l
Two days before surgery the patient was commenced on Platelets 30 ϫ 109/l
intravenous heparin and was continued on a daily APTT 94 s (control 45 s)
infusion of 28,000 units per day for the next 7 days. On INR
the fifth post-operative day she complained of a painful Factor V Leiden 3.8
lower left limb. On examination the lower limb was
swollen and tense. Doppler venography confirmed an mutation Absent
ileo-femoral deep-vein thrombosis. The patient was
commenced on warfarin but developed gangrene of the
skin affecting the toes after 12 hours.

Question 114 Question 115

What is the diagnosis? What is the next management step?
a. Protein S deficiency. a. Stop warfarin and switch to danaparoid.
b. Protein C deficiency. b. Switch to therapeutic dose of low-molecular-
c. Heparin-induced thrombocytopenia type I. weight heparin.
d. Heparin-induced thrombocytopenia type II. c. Give intravenous protein C concentrate.
e. Anti-thrombin III deficiency. d. Give intravenous protein S concentrate.
e. Stop all anticoagulation until platelet count is
>100 ϫ 109/l.

Question 116 116

A patient with psoriasis was referred to the psychiatrist
and found to have manic depression. He was started on
medication that resulted in severe exacerbation of the
rash shown (116).

Which one of the following medications is the most
likely cause for the deterioration?

a. Amitryptilline.
b. Paroxetine.
c. Risperidone.
d. Lithium.
e. Haloperidol.

122

Answers 114 and 115

Answer 114 vein thrombosis, pulmonary embolism, sinus vein
d. Heparin-induced thrombocytopenia type II. thrombosis, myocardial infarction, cerebrovascular
thrombosis and infarction of any other organ. Another
Answer 115 different manifestation is skin necrosis. The condition
a. Stop warfarin and switch to danaparoid. should be suspected in anyone who develops an
abnormally low platelet count or a >50% drop from the
The diagnosis is heparin-induced thrombocytopenia type original platelet count in association with a thrombotic
II, also known as heparin-induced thrombocytopenic episode a few days after starting heparin. The platelet
thrombosis. In contrast with HIT type I, which is a count rarely drops below 20,000 (the usual nadir is
transient isolated thrombocytopenia usually occurring approximately 60,000), therefore bleeding is rare.
after 48 hours of therapy with unfractionated heparin,
HIT type II is an immune-mediated disorder resulting in It is important to note that there are reports of
the formation of antibodies against the heparin-platelet delayed onset HIT type II that may occur nine days after
factor 4 complex. HIT type II has a prevalence of heparin has been stopped.
between 0.3 and 3% of patients who have been on
heparin for between 4 and 10 days. The condition rarely The diagnosis is clinical, being based upon a
occurs after 10 days of treatment. thrombocytopenia in association with heparin therapy
and an associated arterial or venous thrombus. It is
HIT II is characterized by an increased level of IgG confirmed by demonstrating heparin-induced platelet
and IgM antibodies to heparin, which results in both antibodies (serotonin reactive assay, heparin-induced
platelet activation causing venous and arterial thromboses platelet aggregation or solid phase immunoassay).
and thrombocytopenia due to immune-mediated
destruction of platelets. Venous thromboses are more Type II HIT can be prevented by using low-
common than arterial ones. Manifestations include deep- molecular-weight heparin or limiting the use of heparin
to less than five days (i.e switching to warfarin early if
long-term anticoagulation is required).

Management of HIT (type II)

1. Stop heparin.
2. Don’t switch from unfractionated heparin to LMWT heparin once the diagnosis is made (10% cross

reactivity).
3. Danaparoid (heparinoids) or lepirudin (recombinant hirudin) should be started even if the platelet count is

very low unless there is active bleeding. These drugs are the mainstay of therapy for HIT type II.
4. Don’t use warfarin alone if deep-vein thrombosis (can get skin necrosis due to transient acquired protein C

deficiency).
5. Don’t give warfarin until platelet count is >100 ϫ 109/l.

Answer 116

d. Lithium. and antimalarial drugs. Other drugs include ACE
inhibitors and NSAIDs.
The patient has developed severe psoriasis. Psoriasis may
be exacerbated by drugs and infections. With respect to Viral (including HIV) and bacterial infections are
drugs, the commonest culprits are beta-blockers, lithium recognized exacerbating factors for psoriasis.
Streptococcal infections have been particularly implicated
in guttate psoriasis.

Clinical Cases 123

Question 117 1. What is the diagnosis?
a. Multiple myeloma.
A 69-year-old woman presented with pain in her b. Aplastic anaemia.
shoulders and upper back, polydipsia and polyuria. c. Hyperparathyroidism.
d. Chronic renal failure.
Investigations are shown. e. Vitamin D toxicity.

Hb 9 g/dl 2. What is the commonest cause of renal failure in
WCC 14 ϫ 109/l this condition?
Platelets 140 ϫ 109/l a. Hypercalcaemia.
MCV 89 fl b. Amyloid deposition.
Sodium 129 mmol/l c. Tubular obstruction by light chain.
Potassium 3.4 mmol/l d. Analgesic nephropathy.
Urea 26 mmol/l e. Hyperuricaemia.
Creatinine 290 ␮mol/l
Calcium 2.8 mmol/l 3. Give one possible cause for the low potassium.
Phosphate 1.9 mmol/l a. Vomiting.
Total protein 98 g/l b. Diarrhoea.
Albumin 34 g/l c. Diuretic therapy.
d. Proximal renal tubular acidosis.
e. Distal renal tubular acidosis.

Questions 118 and 119

A 14-year-old Asian boy diagnosed with pulmonary presented with malaise, myalgia, nausea and anorexia. On
tuberculosis was commenced on antituberculous drugs examination he was febrile with a diffuse erythematous
consisting of isoniazid, rifampicin, pyrizinamide and rash. The blood pressure was 90/55 mmHg and there
ethambutol. Four days after commencing treatment he were crackles and bronchial breathing audible over the
right upper lobe.
Hb 14 g/dl
WCC 12 ϫ 109/l Investigations are shown.
Neutrophils 7 ϫ 109/l
Lymphocytes 2.5 ϫ 109/l Question 118
Eosinophils 4 ϫ 109/l
Basophils 0.1 ϫ 109/l What is the cause of this patient’s renal dysfunction?
Monocytes 0.4 ϫ 109/l a. Renal tuberculosis.
Platelets 300 ϫ 109/l b. Acute tubular necrosis.
ESR 120 mm/h c. Acute interstitial nephritis.
Sodium d. Acute post-infectious glomerulonephritis.
Potassium 130 mmol/l e. Rhabdomyolysis.
Chloride
Urea 6.5 mmol/l Question 119
Creatinine
Glucose 87 mmol/l What is the most important initial step in his
Urinary microscopy management?
26 mmol/l
24-hour urinalysis 300 ␮mol/l a. Renal biopsy.
9 mmol/l b. Haemodialysis.
c. Corticosteroids.
White blood cells, d. Discontinuation of isoniazid.
e. Discontinuation of rifampicin.
red blood cells and white

cell casts

Sodium 60 mmol/l

Protein 0.8 g

124

Answer 117

1. a. Multiple myeloma. The question tests understanding of the renal and
2. c. Tubular obstruction by light chain. biochemical complications of myeloma.
3. d. Proximal renal tubular acidosis.
The causes of anaemia, renal failure and other
The diagnosis of myeloma is based on the very high biochemical abnormalities in myeloma are covered in the
globulin count (albumin subtracted from total protein answers above. (See also Answer 38.)
content), anaemia, renal failure and hypercalcaemia.

Answers 118 and 119

Answer 118 listed below can produce other forms of acute renal
c. Acute interstitial nephritis. failure.

Answer 119 Genitourinary tuberculosis usually has an insidious
e. Discontinuation of rifampicin. presentation, with dysuria and gross haematuria being the
most common symptoms, and thus not the most likely
The patient presents with fever, rash, eosinophilia and diagnosis in this case. Patients with rhabdomyolysis
renal impairment shortly after commencing anti- typically present with the triad of pigmented granular
tuberculous therapy. The findings are consistent with casts in the urine, pigmenturia due to myoglobinuria and
acute interstitial nephritis secondary to rifampicin. AIN is a marked elevation in the plasma level of creatine kinase.
most commonly due to drug therapy (Table).
Although the clinical picture is often highly suggestive
The onset of AIN after drug therapy ranges from three of AIN, the diagnosis can be confirmed only by renal
to five days following a second exposure, to as long as biopsy. In the absence of severe disease, it is acceptable to
several weeks with a first exposure. However, the latent begin by observing the response to discontinuation of the
period may be as short as one day with rifampicin or as suspected offending drug. No further evaluation or
long as 18 months with an NSAID. therapy is required if renal function begins to improve
within several days.
Patients typically present with an acute rise in the
plasma creatinine concentration temporally related to an Indications for biopsy generally include uncertainty as
offending drug or infection. Fever occurs in most cases to the diagnosis, advanced renal failure or lack of
and may be accompanied by a rash. The urine sediment spontaneous recovery following cessation of drug
usually reveals white cells, red cells, and white cell casts. therapy. An alternative – particularly in those who are
Eosinophilia and eosinophiluria are present in over 75% poor candidates for renal biopsy – is to initiate a trial of
of cases with the exception of disease induced by NSAIDs corticosteroids (such as 1 mg/kg of prednisone per day).
where fever, rash, and eosinophilia are typically absent. Patients with AIN typically begin to improve within one
Most patients have normal or only mildly increased to two weeks and rapidly return to their baseline plasma
protein excretion (<1 g/day) although some older creatinine concentration.
individuals have significant proteinuria (approximately 3
g/day). Concurrent nephrotic syndrome due to minimal Causes of acute tubulo-interstitial nephritis
change disease is often seen with NSAIDs. Signs of
tubulo-interstitial damage, such as the Fanconi syndrome Drug-induced
and renal tubular acidosis, may also be present. • Non-steroidal anti-inflammatory drugs
• Penicillins
The diagnosis should be suspected from the temporal • Cephalosporins
relation to an offending drug and the characteristic • Sulphonamides
laboratory findings. The urinary findings usually • Loop and thiazide-type diuretics
distinguish AIN from other causes of acute renal failure
such as glomerulonephritis and acute tubular necrosis. Non drug-induced
Acute glomerulonephritis is usually associated with the • Sarcoidosis
presence of red cell casts in the urine. In acute tubular • Legionella infection
necrosis urinalysis typically shows granular and epithelial • Leptospirosis
cell casts and free epithelial cells. Even when none of • Streptococcal infection
these other conditions appears to be present, examination
of the urine is still important because some of the drugs

Clinical Cases 125

Question 120

A 56-year-old male was admitted with weight loss and
fatigue. He consumed 40 units of alcohol per week. On
examination he was pale and had a palpable spleen 10 cm
below the costal margin.

Investigations are shown.

Hb 8 g/dl What is the diagnosis?
WCC 23 ϫ 109/l a. Myelofibrosis.
Platelets 500 ϫ 109/l b. Polycythaemia rubra vera.
MCV 90 fl c. Myelodysplasia.
Neutrophils 58% d. Chronic myeloid leukaemia.
Lymphocytes 2% e. Hypersplenism secondary to cirrhotic portal
Monocytes 1% hypertension.
Eosinophils 1%
Basophils 2%
Neutrophil alkaline 25 iu/l
(NR 35–100 iu/l)
phosphatase

Question 121 What is the next step in his management?
a. Echocardiography prior to discharge.
A 67-year-old man presented with central chest pain b. Coronary angiography prior to discharge.
associated with ST segment depression in the inferior and c. Exercise stress test prior to discharge.
lateral leads. He had smoked 20–30 cigarettes per day for d. CRP.
over 40 years. Troponin T 12 hours after admission was e. Discharge home and arrange outpatient exercise
0.4 ng/l (NR <0.05 ng/l). The patient remained pain test in six weeks.
free for three days and the ECG changes resolved. He
was treated with antiplatelet agents, a statin, ACE
inhibitor and a beta-blocker.

Question 122 What is the management of the patient?
a. Consider termination of pregnancy.
A 24-year-old woman presented with a swollen left lower b. Start subcutaneous fractionated heparin and
limb when she was eight weeks pregnant. Lower limb continue for the entire pregnancy,
vein Doppler studies confirmed a femoral vein deep-vein maintaining an antifactor Xa >1.0 (four hours
thrombosis. There was no family history of after injection). After pregnancy give
thrombophilia. warfarin for six weeks.
c. Start fractionated heparin initially and switch to
warfarin in the second trimester. Continue
warfarin until the middle of the third trimester
and then switch patient back to heparin until
term.
d. Start warfarin and continue throughout
pregnancy.
e. Use high-dose aspirin during pregnancy and
switch to warfarin for six weeks post-partum.

126

Answer 120

d. Chronic myeloid leukaemia. from myelofibrosis by karyotyping for Philadelphia
chromosome, measurement of neutrophil leucocyte
The patient has large splenomegaly and a high white cell alkaline phosphatase level and bone marrow analysis.
count consisting predominantly of neutrophils. The Patients with CML usually have the Philadelphia
differential diagnosis is between CML and myelofibrosis. chromosome abnormality and low neutrophil alkaline
Both conditions may be associated with much higher phosphatase levels. Further support for the diagnosis of
white cell counts; however, CML can be differentiated CML in this question is the relatively high basophil
count. (See Question 392.)

Answer 121

b. Coronary angiography prior to discharge. in patients with non-ST elevation acute coronary
syndrome was associated with a lower rate of adverse
The patient has chest pain and raised troponin in the cardiac events in the ensuing nine months. Both drugs
absence of ST elevation. By definition this patient has are initiated on admission. Aspirin is continued
suffered a non-ST elevation myocardial infarction. Such indefinitely whereas clopidogrel should be continued for
patients have a relatively low risk of in-hospital mortality up to 12 months. Glycoprotein IIb/IIIa inhibitors are
compared with ST segment elevation myocardial started at the same time as antithrombotic therapy and
infarction; however, six-month mortality exceeds that of subcutaneous fractionated heparin in all patients with
patients with STEMI and is around 20%. Therefore, it is NSTEMI and in patients with unstable angina who
prudent that all appropriate patients with NSTEMI have continue to have pain and/or dynamic ECG changes
coronary angiography and revascularization (if required) representative of myocardial ischaemia. Eptifibatide and
prior to discharge from hospital. The same applies to tirofiban are the most commonly used IIb/IIIa
patients who present with chest pain and marked ST glycoprotein receptor inhibitors. IIb/IIIa glycoprotein
segment depression even if the serum cardiac troponin is receptor blockers are continued for up to 72 hours, by
not elevated (Table A). which time most patients should have undergone
coronary angiography.
In the interim the management of both groups of
patients is essentially the same and consists of anti- Table B Management of non-ST elevation
thrombotic agents (aspirin and clopidogrel together), myocardial infarction
fractionated heparin, and IIb/IIIa platelet receptor
blocking agents (Table B). High-dose statins may play a 1. Relieve pain as in ST elevation myocardial
role in plaque pacification in the peri-infarction period. infarction
In addition beta-blockers may reduce myocardial oxygen
demand. GTN infusion is useful in reducing symptoms of 2. Aspirin loading dose of 300 mg followed by
acute myocardial ischaemia. 75–150 mg daily indefinitely

The benefits of aspirin in myocardial infarction are 3. Clopidogrel loading dose 300 mg followed by
well established. Over the past few years the CURE study 75 mg daily for 12 months
demonstrated that the addition of clopidogrel to aspirin
4. Fractionated heparin, usually enoxaparin
Table A Indications for in-patient coronary (subcutaneously), until coronary angiography.
angiography in patients with non-ST elevation
coronary syndromes 5. IIb/IIIa glycoprotein receptor blockers for up
to 72 hours. Ideally most patients should have
• Raised serum troponin T or I had coronary angiography within 72 hours of
• Chest pain with dynamic ST segment changes presentation

of myocardial ischaemia 6. Initiate high-dose statin therapy
• Chest pain with clinical evidence of left 7. Beta-blocker therapy on admission as with ST

ventricular dysfunction elevation myocardial infarction
• Chest pain associated with malignant ventricular 8. Coronary angiography in all patients where

arrhythmias general health and comorbidities allow for the
procedure to be performed safely
9. ACE inhibitor prior to discharge

Clinical Cases 127

Early use of high-dose statins, such as simvastatin It is unclear whether early initiation of an ACE
40 mg or atorvastatin 40–80 mg, has been shown to inhibitor alters the immediate prognosis in patients with
reduce adverse cardiac events 30 days after the initial NSTEMI unless the patient has left ventricular
insult. There are no trials assessing the effect of beta- dysfunction. However, the long-term benefits of ACE
blockers in NSTEMI; however, given the magnitude of inhibitor therapy in patients with coronary artery disease
prognostic benefit of this group of drugs in patients with are now well established (EUROPA and HOPE studies)
STEMI, it seems reasonable to deduce that they are and it is recommended that all patients presenting with
probably effective in the management of patients with NSTEMI are initiated on an ACE inhibitor prior to
NSTEMI. discharge home, provided there are no contraindications.

Answer 122

b. Start subcutaneous fractionated heparin and 2. Continue weight-adjusted low-molecular weight
continue for the entire pregnancy, maintaining an heparin aiming for a 4-hour post injection antifactor
antifactor Xa >1.0 four hours after injection of Xa of 1.0 u. Antifactor Xa levels need to be measured
heparin. After pregnancy give warfarin for six once or twice a month as the patient is gaining
weeks. weight. This is currently the anticoagulant strategy of
choice.
The patient requires anticoagulation but warfarin is
contraindicated in the first 13 weeks of pregnancy as it is 3. Start with either unfractionated or low-molecular
teratogenic (skeletal, cartilage and foetal CNS weight heparin during the first trimester and then
abnormalities). Heparin, on the other hand, does not switch to warfarin until the mid-third trimester
cross the placenta and has not been associated with (maintenance INR of 2). During the mid-third
teratogenic effects. The recommendation is to use either trimester the patient should be switched back to
unfractionated or low-molecular weight heparin, at least heparin until term because warfarin is associated with
in the first 13 weeks. a higher risk of foetal haemorrhage during vaginal
delivery and the baby’s INR cannot be reversed by
Following this there are three options, as follows: giving the mother fresh-frozen plasma during labour.

1. Continue dose-adjusted unfractionated heparin In all three cases above the patient should be
throughout pregnancy, maintaining an APPT twice anticoagulated with warfarin for six weeks following
that of the normal range. However, treatment with delivery, as these women have low antithrombin III
unfractionated heparin requires daily monitoring of levels, increase in factors I, II, VII, VIII and X and have
the APTT, which is inconvenient and impractical. increased venous stasis.
Long-term unfractionated heparin injections are
associated with a high risk of osteoporosis. Aspirin is not effective in preventing propagation of a
venous thrombus or preventing pulmonary embolism in
pregnancy.

Question 123 What is the treatment for the hypertension?
a. Continue lifestyle modification for another six
A 70-year-old obese male saw his GP and was noted to months.
have a blood pressure of 170/98 mmHg. The fundi were b. Bendroflumethiazide (bendroflumethazide).
normal. Urinalysis was normal. Urea, electrolytes and c. Ramipril.
blood sugar were normal. The total cholesterol measured d. Atenolol.
6 mmol/l. The patient was advised to lose weight and e. Doxazocin.
adhere to a low-salt diet. Blood pressure readings over
the next six months were 170/96 mmHg, 168/96
mmHg and 166/96 mmHg.

128

Answer 123

b. Bendroflumethiazide (bendroflumethazide). require two or even three antihypertensive drugs to help
control blood pressure. All of these groups of drugs have
The patient has moderate to severe hypertension and been shown to reduce cardiovascular mortality and
requires lifestyle modification advice as well as stroke. The aim is to bring down the blood pressure to
pharmacological therapy to help control his blood ≤140/90 mmHg. In patients with diabetes, stroke,
pressure. Hypertension in the elderly is best treated with coronary disease, heart failure or nephropathy the goals
calcium channel blockers or a thiazide diuretic. The use are more stringent and the aim is to reduce blood
of thiazide diuretics as first line in most elderly patients pressure to ≤130/85 mmHg.
with hypertension is supported by the recently published
ALLHAT study, which was the largest study ever in the During initiation of antihypertensive drugs, the ABCD
hypertensive population. The study showed that thiazide method of prescription is useful for stepwise management
diuretics were more effective than beta-blockers, calcium of hypertension. The system is based on the fact that
channel blockers and angiotensin-converting enzyme most young patients (aged ≤55 years) and non-black
inhibitors at reducing cardiovascular mortality in patients have high plasma renin levels and
diabetics and non-diabetics. respond best to drugs primarily targeting the
renin–angiotensin–aldosterone system when used as
All patients should receive advice regarding lifestyle monotherapy. These patients respond to ACE inhibitors,
modification to help reduce the blood pressure (Table A). Angiotensin receptor blockers, or Beta-blockers as
Lifestyle modification alone without antihypertensive monotherapy (i.e. the A and B drugs). In contrast older
drug therapy for a period of six months to one year is patients and black patients (Afro-Caribbean in origin)
reserved for patients with mild hypertension (BP respond better to drugs promoting natriuresis and
140–159/ 90–99 mmHg) who do not have any evidence reducing plasma volume such as Calcium channel
of end-organ damage such as retinopathy, abnormal renal blockers and Diuretics (i.e. the C and D drugs) when
function, proteinuria, or any history of diabetes, used as single agents (Table B). Addition of an A or B
nephropathy, stroke, coronary artery disease or heart drug to a patient whose blood pressure is still elevated
failure (Table A). Most patients with hypertension while taking a C or D drug and vice versa has an additive
require antihypertensive drug therapy. effect because in combination, these drugs counteract
different mechanisms responsible for high blood pressure.
The British Hypertension Society recommends that
any one of the five major classes of antihypertensive drugs This particular system should not be applied to every
(thiazides, beta-blockers, calcium channel blockers, ACE case as there may be compelling indications to use A or B
inhibitors and angiotensin II receptor blockers) may be class drugs in patients aged >55 years or black patients if
used as first-line monotherapy, although most patients there is a history of diabetes (ACE inhibitor or A II
receptor blocker) or heart failure (all of the A or B drugs).

Table A Lifestyle Table B The stepwise management of hypertension using
modification the ABCD system

• Lose weight to bring Young Age ≥55 years
BMI down below 30 Non-black patients Black patients

• Reduce alcohol Step 1 A or B C or D
consumption Step 2 A or B + C or D
Step 3
• Regular exercise for Step 4 A CD
30–45 min three times
per week

• Reduce salt intake to
6 g/day

• Increase potassium to
90 mmol/day

• Smoking cessation

A CD
+

Spironolactone or doxazocin

Clinical Cases 129

Question 124 124

A 70-year-old male presented with a four-week history of
increasing dyspnoea and swollen ankles. There was no
evidence of ischaemic heart disease, diabetes or alcohol
abuse. On examination he had clinical and radiological
evidence of pulmonary oedema. The blood pressure
measured 180/100 mmHg. He was treated with
intravenous furosemide with good results.

Investigations are shown.

Hb 10 g/dl

WCC 10 ϫ 109/l

Platelets 200 ϫ 109/l

Sodium 139 mmol/l

Potassium 5.1 mmol/l

Urea 14.9 mmol/l

Creatinine 250 ␮mol/l

Albumin 29 g/l

Chest X-ray after diuretic treatment (124)

ECG Sinus rhythm; What is the most probable cause for the heart failure?
a. Hypertensive heart disease.
small complexes; b. Anaemia.
c. Ischaemic heart disease.
Q waves inferior and d. Cardiac amyloid.
e. Renal artery stenosis.
lateral leads

Urinalysis Protein ++

Renal ultrasound Normal kidneys

Question 125 What is the most effective initial pharmacological step
in reducing cardiovascular mortality?
A 54-year-old non-insulin-dependent diabetic patient
was referred to the blood pressure clinic for assessment a. Weight loss.
and control of blood pressure. He was a non-smoker. b. Increase dose of statin.
He had a body mass index of 28. The blood pressure in c. Start atenolol.
clinic measured 180/98 mmHg on three successive d. Start bendroflumethiazide.
occasions. His medication comprised metformin 1 g e. Start losartan.
twice daily, simvastatin 40 mg od, aspirin 75 mg and
amlodipine 10 mg od. Renal function was normal but
urinalysis revealed proteinuria +. The HbA1c was 7%.
The total cholesterol was 5.2 mmol/l. The 12-lead
ECG revealed voltage criteria for left ventricular
hypertrophy.

130

Answer 124

d. Cardiac amyloid. Bone involvement is characterized by expansile lytic
lesions, osteopenia and pathological fractures. Osteolytic
The commonest causes of heart failure in the UK are lesions are thought to be due to stimulation of
ischaemic heart disease and hypertension. However, the osteoclastic activity. The vertebral bodies, skull thoracic
clue to the answer in this question is in the interpretation cage, pelvis and proximal aspects of the humeri and
of the chest X-ray, which shows expansile lytic lesions in femora are the commonest sites of bone lesions. The
the ribs consistent with the diagnosis of multiple most serious complication of bone involvement is cord
myeloma. A significant proportion of patients with compression from vertebral body collapse.
multiple myeloma develop amyloidosis. Cardiac amyloid
is associated with restrictive dilated cardiomyopathy, Technetium bone scans, which primarily detect
conduction disturbance and angina due to amyloid osteoblastic activity, are not as useful as conventional
deposits in the coronary arteries. The characteristic plain radiographs (skeletal survey) at identifying the
features of cardiac amyloid on the 12-lead ECG include extent of bone involvement in multiple myeloma.
small complexes, pseudo-infarcts (q-waves) and heart
block (see Question 389). Amyloidosis commonly causes All patients with multiple myeloma should be treated
renal failure in multiple myeloma as in this case (see with bisphosphonates to reduce the risk of skeletal
Question 38). complications. Treatment with bisphosphonates reduces
bone pain, and the rate of pathological fractures and cord
Almost 80% of patients have evidence of bone compression. A small proportion of patients on
involvement from myeloma at the time of diagnosis. bisphosphonates for myeloma may develop nephrotic
syndrome due to focal segmental glomerulosclerosis.

Answer 125

e. Start losartan. target blood pressure levels (resting BP
<140/90 mmHg). The initial and subsequent doses of
The most effective method of reducing cardiovascular both losartan and atenolol were 50 and 100 mg/day,
mortality in this patient with non-insulin-dependent respectively. The fall in blood pressure was similar in the
diabetes would be to control the blood pressure two groups (30/17 mmHg).
effectively. Weight loss alone in a patient with this
magnitude of hypertension would not be enough to The study showed that losartan was more effective
reduce blood pressure. than atenolol in reducing mortality, primarily owing to a
reduction in fatal or non-fatal stroke. The benefit from
While all groups of antihypertensive drugs listed have losartan was more pronounced among diabetics com-
been shown to be equally efficacious in bringing the pared to non-diabetics. The benefit was most prominent
blood pressure down in 40–60% of patients when used as in diabetic patients who had not previously received anti-
monotherapy, there are clinical settings in which certain hypertensive therapy.
groups of drugs may have additional benefits on
cardiovascular mortality. In patients with non-insulin- The recently published ALLHAT study, which was the
dependent diabetes mellitus, both angiotensin II receptor largest study ever in the hypertensive population, showed
blockers and angiotensin-converting enzyme inhibitors that thiazide diuretics were more effective than beta-
have been particularly effective in reducing cardiovascular blockers, calcium channel blockers and angiotensin-
mortality, as shown in the HOPE study and LIFE converting enzyme inhibitors at reducing cardiovascular
studies, respectively. mortality in diabetics and non-diabetics. However,
patients in the ALLHAT study did not have severe
The LIFE trial evaluated the specific cardiovascular hypertension and not all had ECG criteria for left
benefits of losartan in high-risk patients with moderate ventricular hypertrophy. Furthermore, the ALLHAT
to severe hypertension (resting BP 160–200/ study did not evaluate angiotensin receptor blockers
95–115 mmHg) and electrocardiographic evidence of specifically. In this particular case one would add up to
left ventricular hypertrophy. 100 mg losartan initially and introduce a thiazide diuretic
if the BP was still >140/90 mmHg.
Patients were randomly assigned to either losartan or
atenolol therapy, with dose increases and the addition of The patient will also require additional cholesterol
hydrochlorthiazide (as well as other agents) to attain the lowering therapy in the form of a statin; however, in this
case BP reduction is the priority.

Clinical Cases 131

Question 126

A 69-year-old male was seen in the renal clinic On examination he appeared pale. There was no
complaining of fatigue and dyspnoea. He had chronic evidence of oedema. The heart rate was 90 beats/min and
renal impairment secondary to focal segmental regular. The blood pressure measured 140/80 mmHg.
glomerulonephritis and had been on haemodialysis for six The JVP was not raised. On examination of the
months. The patient was noted to have been anaemic precordium the apex was not displaced. Auscultation of the
three months ago for which he was commenced on oral heart was normal and the chest was clear.
iron supplements. He had controlled hypertension but
no other medical history. Medication comprised Investigations are shown.
amlodipine 10 mg od, ferrous sulphate 200 mg bd and
simvastatin 40 mg od.

Hb 9.9 g/dl How would you treat the anaemia?
WCC 5 ϫ 109/l a. Transfuse two units of packed red cells.
Platelets 200 ϫ 109/l b. Increase dose of oral iron therapy.
MCV 88 fl c. Intravenous iron therapy.
Sodium 138 mmol/l d. SC erythropoietin.
Potassium 5.1 mmol/l e. IV iron and SC erythropoietin.
Urea 20 mmol/l
Creatinine 340 ␮mol/l
Calcium 2.0 mmol/l
Phosphate 1.3 mmol/l
Albumin 38 g/l
Glucose 4 mmol/l
Iron 9 mmol/l (NR 14–32 mmol/l)
TIBC 50 mmol/l (NR 40–80 mmol/l)
Ferritin 36 mg/l (NR 15–250 mg/l)
ECG Left ventricular hypertrophy
Chest X-ray Slight cardiac enlargement;

clear lung fields

Question 127

A 17-year-old boy was referred to the local paediatrician measured 1.73 m and had already started shaving. There
because he had not grown very much in the past four years. were no other siblings. Both parents were well. The father
Apart from feeling more lethargic than his school friends, was 1.76 m tall and the mother was 1.63 m tall.
he gave no other history of note. He measured 1.53 m. He
had never needed to shave. His younger brother, aged 16, On examination, he was slightly pale. He did not have
any facial hair. He was below the third centile for height
Hb 10 g/dl and was just at the 10th centile for weight. He did not
WCC 7 ϫ 109/l have any evidence of secondary sexual characteristics, and
Platelets 200 ϫ 109/l his testes were small. The blood pressure was
MCV 102 fl 115/70 mmHg.
Sodium 138 mmol/l
Potassium 4.3 mmol/l Investigations are shown.
Urea 4 mmol/l
Calcium 2.1 mmol/l List two possible diagnoses.
Phosphate 1.1 mmol/l a. Coeliac disease.
Albumin 38 g/l b. Growth hormone deficiency.
Glucose 4.3 mmol/l c. Constitutional delay in puberty.
d. Hypothyroidism.
e. Vitamin D resistant hypophosphataemic rickets.

132

Answer 126

c. Intravenous iron therapy. chronic renal failure. The patient in question has
evidence of absolute iron deficiency and requires iron
The patient has chronic renal failure and is anaemic. supplements before considering the need for EPO.
Anaemia is a common complication of chronic renal Patients with iron deficiency usually require intravenous
failure. The cause of anaemia in CRF is multifactorial but iron supplements to replenish iron stores, particularly
deficiency of erythropoietin plays an important role. patients receiving haemodialysis. A treatment algorithm
Anaemia is associated with a reduced quality of life and for anaemia in chronic renal failure is shown below.
worse clinical outcomes as well as a higher prevalence of Patients who are not iron deficient generally respond to
left ventricular hypertrophy and left ventricular cavity size erythropoietin.
in patients with chronic renal failure.
Causes of anaemia in renal failure
Anaemia is defined as an Hb <11g/dl in pre-pubertal
children and pre-menopausal women and an Hb • Erythropoietin deficiency
<12 g/dl in adult males and post-menopausal women. • Chronic blood loss (iron deficiency)
Patients with chronic renal failure should be maintained • Hypothyroidism
on an Hb of at least 11–12 g/dl. • Vitamin B12 or folate deficiency
• Chronic infection or inflammation
Other causes of anaemia, including iron deficiency, • Aluminium toxicity
should be excluded before attributing the anaemia to • Malignancy
chronic renal failure (Table). Iron stores should be • Haemolysis
replenished before considering therapy with • Myeloma
erythropoietins. In patient with chronic renal failure, the • Bone marrow infiltration
most useful markers of iron stores are the serum ferritin • Marrow aplasia
and the transferrin saturation which = Fe/TIBC ϫ 100.
A serum ferritin <100 mg/l and a TSAT ≤20% is
indicative of absolute iron deficiency in a patient with

Treatment algorithm for anaemia in chronic renal failure
Haemoglobin <11 g/dl

Serum ferritin <100 mg/l Haematinics screen Treat with 200 mg iron
Ϯ Yes sucrose weekly every three

TSAT <20% weeks

No Check haematinics one
month post-IV iron
Start EPO No
300 iu/week Serum ferritin <100 mg/l
Ϯ
Check monthly
response TSAT >20%

Achieve and Yes
maintain target Hb
Hb <11 g/dl
Commence EPO

Clinical Cases 133

Answer 127

a. Coeliac disease. explain the short stature, delayed puberty and
macrocytosis; however, the problem has been present for
This is a difficult question. There is evidence of short approximately four years and one would expect a history
stature, delayed puberty and a macrocytosis. These three of performing poorly at school or the presence of dry skin
features can be best explained by malabsorption. The on examination. In addition, many children with
commonest cause of malabsorption in the Western world hypothyroidism are relatively overweight in relation to
is coeliac disease. The macrocytosis in this respect is due their height.
to folate deficiency. It is unusual, however, not to also
have biochemical evidence of osteomalacia. The GH deficiency or gonadotrophin deficiency could
differential diagnosis here is hypothyroidism, which may explain short stature and delayed puberty, but they do
not explain the macrocytosis. As with hypothyroidism,
children with GH deficiency are relatively plump.

Question 128 What is the diagnosis?
a. Septic cavernous sinus thrombosis.
A 15-year-old male presented with a four-day history of b. Viral encephalitis.
severe right-sided headaches affecting the orbit and right c. Bacterial meningitis.
maxillary area. On examination he had acneform lesions d. Cerebral abscess.
around the nose and cheeks. Shortly after admission he e. Cogan’s syndrome.
became drowsy and developed a high temperature. He
complained of diplopia. Subsequent physical examination
revealed swelling of the right eye, a partial ptosis of the
right eye and a lateral gaze palsy affecting the same eye.
Fundoscopy revealed papilloedema. There was no
evidence of nuchal rigidity.

Question 129

A 60-year-old man presents with general malaise and
drowsiness. On examination, the blood pressure was
120/70 mmHg. Investigations are shown.

Sodium 108 mmol/l What is the diagnosis?
Potassium 4 mmol/l a. Addison’s disease.
Urea 3 mmol/l b. Hypothyroidism.
Creatinine 56 ␮mol/l c. SIADH secretion.
Bicarbonate 23 mmol/l d. Ectopic ACTH-secreting tumour.
Random blood sugar 4.2 mmol/l e. Gastric outlet obstruction.
Urine glucose Not detected
Urine ketones Not detected

134

Answer 128

a. Septic cavernous sinus thrombosis. intravenous heparin. Corticosteroids may be used (after the
infection has been cleared with antibiotics) to improve
The patient has septic dural sinus thrombosis syndrome. inflammation around the cranial nerves. Corticosteroids are
Three main types of dural sinus thrombosis are mandatory if pituitary infarction, a recognized
recognized, notably cavernous sinus thrombosis, lateral complication of cavernous sinus thrombosis, occurs.
sinus thrombosis and superior saggital sinus thrombosis.
Lateral sinus thrombosis is rare and almost always
The commonest variety is the cavernous sinus complicates otitis media. It is characterized by fifth and
thrombosis syndrome, possibly because the facial veins drain sixth nerve palsy and papilloedema. It may be
into the sinus and the most common source of infection is complicated by hydrocephalus. Treatment is as above but
due to squeezing of nasal furuncles without antibiotic a radical mastoidectomy is also necessary.
cover. Other sources of infection include otitis media,
sinusitis and dental infections (Table). The commonest Superior saggital venous thrombosis may complicate
organism implicated is Staphylococcus aureus. Patients with sinusitis. Thrombosis of the anterior segment of the sinus
cavernous sinus thrombosis present with severe peri-orbital causes headaches, which resolve on the development of
headache, which also affects areas innervated by the collaterals. Complete thrombotic occlusion of the
ophthalmic and maxillary branches of the trigeminal nerve. superior saggital sinus is invariably fatal, resulting in large
Fever and peri-orbital oedema usually develop afterwards. venous cortical infarction.
Ocular swelling, chemosis, ophthalmoplegia and drowsiness
are recognized complications. Ophthalmoplegia is due to Cogan’s syndrome is a chronic inflammatory disorder
compression of the third, fourth and sixth cranial nerves. characterized by interstitial keratitis and vestibulo-
Headache and associated ophthalmoplegia should always auditory dysfunction, which may be associated with a
alert the clinician to the possible diagnosis of cavernous systemic vasculitis.
sinus thrombosis.
Causes of dural sinus thrombosis
The diagnosis of cavernous sinus thrombosis is either
with high resolution CT scan of the orbit with contrast or • Infections of the face, ear and sinuses
a gadolinium-enhanced MRI scan of the orbit. • Hereditary thrombophilic states
Management comprises intravenous flucloxacillin and • Hyperviscocity states
• Oral contraceptive pill/pregnancy
• Behçet’s disease

Answer 129 Causes of SIADH

c. SIADH secretion. Central nervous system
• Head injury
Syndrome of inappropriate ADH secretion (see Table for • Encephalitis/meningitis
causes) is the most likely diagnosis when the serum • Cerebral abscess
sodium is <115 mmol/l. Hypothyroidism can cause • Cerebral neoplasm
failure to excrete water, and produce a similar • Cerebral haemorrhage
biochemical picture to SIADH. The diagnosis is
confirmed by demonstrating a low plasma osmolality and Respiratory system
an inappropriately high urine osmolality. • Bronchial carcinoma (particularly oat cell)
• TB
• Pneumonia (Legionnaire’s is the college favourite)
• Empyema

Drugs
• Chlorpropamide
• Chlorpromazine
• Carbamazepine
• Opiates
• Vincristine

Miscellaneous
• Guillain–Barré syndrome
• Acute intermittent porphyria
• Carcinoma pancreas/thymoma

Clinical Cases 135

Questions 130–132 Question 130

A 32-year-old woman who was 12 weeks into her first What is the most probable cause for the hypertension?
pregnancy presented with headache and malaise. Her a. Atheromatous left-sided renal artery stenosis.
blood pressure in the antenatal clinic four weeks b. Fibromuscular dysplasia of the left renal artery.
previously was 120/70 mmHg. The current blood c. Unilateral reflux nephropathy affecting left
pressure reading was 220/112 mmHg. kidney.
d. Congenital atrophic kidney.
Investigations are shown. e. Pre-eclampsia.

Hb 13 g/dl Question 131
WCC 7 ϫ 109/l
Platelets 180 ϫ 109/l If the patient has papilloedema what is the drug
ESR 12 mm/h treatment of choice?
Sodium 140 mmol/l
Potassium 4 mmol/l a. IV labetolol.
Urea 6 mmol/l b. Oral methyldopa.
Creatinine 110 ␮mol/l c. Oral bendroflumethiazide.
Autoantibody screen Normal d. Oral nifedipine.
ECG Normal e. IV nitroprusside.
Urinalysis Normal
Renal ultrasound 12 cm kidney on the
right and a 7 cm
kidney on the left

Question 132

Which one of the following drugs should not be used
to treat hypertension in pregnancy?

a. Labetolol.
b. Methyldopa.
c. Lisinopril.
d. Nifedipine.
e. Hydralazine.

Question 133 What is the next management step?
a. Start ferrous sulphate.
A 22-year-old male was admitted to hospital after an b. Start lansoprazole.
episode of haematemesis preceded by profuse vomiting. c. Allow home.
The patient had been on an alcoholic binge drinking d. Coeliac axis angiogram prior to discharge.
session 12 hours previously. On examination his heart e. Technetium scan for Meckel’s diverticulum.
rate was 90 beats/min. The blood pressure was
100/60 mmHg. The Hb was 15 g/dl. An upper
gastrointestinal endoscopy performed 24 hours after
admission was normal.

136

Answers 130–132

Answer 130 eclampsia. Pre-eclampsia may not be associated with any
b. Fibromuscular dysplasia of the left renal artery. symptoms, but in severe cases women may present with
headache, papilloedema and abdominal pain due to liver
Answer 131 oedema or hepatic haemorrhage. The management of
a. IV labetolol. hypertension in pre-eclampsia does not halt the
progression of the disease but does reduce the risk of
Answer 132 complications such has cerebral haemorrhage. Delivery is
c. Lisinopril. the definitive method of treating the progression of pre-
eclampsia but the usual practice is to allow the pregnancy
The onset of severe hypertension within a few weeks of to proceed as long as possible with control of
pregnancy in the setting of unilateral kidney disease is hypertension without unduly risking the mother or the
highly suggestive of renal artery stenosis. Two distinct foetus. The management of hypertension in pre-
forms of renal artery stenosis are recognized, notably eclampsia and all other causes of hypertension in
atheromatous renal artery disease, which is more common pregnancy is outlined below.
in middle-aged males with risk factors for atherosclerosis,
and fibromuscular dysplasia, which usually occurs in In the presence of severe symptoms or evidence of
younger females and is characterized by narrowing of the end-organ damage such as heart failure, renal failure or
distal main renal artery or the intrarenal arteries. raised intracranial pressure, the management of severe
hypertension (BP >170/110 mmHg) includes either IV
The most probable diagnosis in this particular case is hydralazine or labetolol. Even in the absence of end-
fibromuscular dysplasia affecting the left renal artery. organ damage or symptoms there is evidence that failure
Affected patients may present with severe exacerbations to treat severe hypertension increases the risk of cerebral
of an otherwise relatively stable (normal or slightly haemorrhage and should be treated. IV nitroprusside is
elevated) blood pressure. In pregnancy, for example, the contraindicated in pregnancy.
increase in blood volume results in increased vascular
oxidative stress in the kidney in patients with renal artery In patients who do not have severe hypertension, the
stenosis, which is a stimulus for increased production of drug of choice is methyldopa. Second-line agents are
angiotensin II, a potent vasoconstrictor. Increased hydralazine and nifedipine, which should be added if the
angiotensin II levels have the effect of large increases in blood pressure is not adequately controlled with
blood pressure and precipitation of left ventricular failure methyldopa. Third-line agents include alpha- and beta-
or encephalopathy. blockers (labetolol and oxprenolol). Beta-blockers are
generally avoided in the first half of pregnancy as they are
While unilateral pyelonephritis is a recognized cause of associated with foetal growth retardation. Thiazide
unilateral small kidney, it does not usually result in diuretics may be used to treat hypertension in women
disturbed renal function unless there is an intrinsic with pre-existing hypertension but should be avoided in
abnormality affecting the contralateral kidney. patients with pre-eclampsia, as many such patients are
volume depleted.
Pre-eclampsia usually occurs after 20 weeks’ gestation,
complicates 5–6% of all pregnancies and is much more ACE inhibitors and possibly also angiotensin II
common in women with pre-existing hypertension. It is receptor blockers are contraindicated in pregnancy.
defined as hypertension and proteinuria (>0.3 g/ Although there is no evidence that they are teratogenic,
24 hours). Up to 2% of patients with pre-eclampsia their use has been associated with foetotoxic effects such
develop seizures and the condition is then termed as growth retardation.

Answer 133

c. Allow home. Patients with an actively bleeding Mallory–Weiss tear
at endoscopy should be treated with endoscopic
Patients with a bleeding Mallory–Weiss tear, but without haemostasis (either adrenaline injection or
risk factors for re-bleeding, clinical features indicating thermocoagulation). They should be hospitalized for at
severe bleeding, or active bleeding at endoscopy will least 48 hours because re-bleeding generally occurs
almost always heal spontaneously. They can be managed within 24 hours of the initial therapeutic endoscopy.
medically with a brief hospitalization of 24 hours.

Clinical Cases 137

Question 134 Which alternative treatment would you use to treat
the osteoporosis?
A 48-year-old post-menopausal woman with asthma was
taking hormone replacement therapy to prevent a. 1,25-Dihydrocholecalciferol.
progression of osteoporosis, which was thought to be b. Calcitonin.
due to steroid therapy. While on hormone replacement c. Alendronate.
therapy she developed a deep-vein thrombosis. d. Raloxifene.
e. Vitamin D and calcium supplements.

Question 135 Which of the following statements is correct?
a. Aggressive management of hypertension will
A 38-year-old man was recently diagnosed with reduce the rate decline of renal function.
polycystic kidney disease after being investigated for b. There is a high chance that the patient will be
hypertension. His father and brother also had dialysis-dependent by the age of 60 years.
hypertension but had never been screened for polycystic c. The patient should undergo magnetic resonance
kidney disease. His paternal uncle had died from heart angiography of the brain to exclude berry
failure. There was no family history of sudden death or aneurysms.
cerebral haemorrhage. His blood pressure measured d. Angiotensin-converting enzyme inhibitor
150/94 mmHg. therapy is beneficial in retarding the rate of
decline of renal function.
e. Renal failure is the commonest cause of death
in this condition.

Question 136

A 20-year-old male was admitted with hypertension and was clear. There was mild ankle swelling. Inspection of
malaise. His blood pressure on admission was the fundi revealed grade I hypertensive retinopathy.
210/124 mmHg. The mini-mental test score was
10/10. Both heart sounds were normal and the chest Investigations are shown.

Hb 11 g/dl What is the immediate management step?
WCC 12 ϫ 109/l a. Haemodialysis.
Platelets 120 ϫ 109/l b. IV actrapid and dextrose 50%.
ESR 60 mm/h c. IV labetolol.
Sodium 138 mmol/l d. IV calcium gluconate.
Potassium 7.4 mmol/l e. High-dose IV furosemide.
Urea 28 mmol/l
Creatinine 500 ␮mol/l
Glucose 4 mmol/l
24-hour urinalysis Volume 400 ml
Protein 1.4 g
Blood ++

138

Answer 134

c. Alendronate. oestrogen – does not promote endometrial hyperplasia or
vaginal bleeding. It may reduce the risk for breast cancer
All patients with or at risk of osteoporosis should have a but the risk of venous thromboembolism is similar to
diet containing 800 iu of vitamin D and take that of oestrogens; therefore, the drug should not be
supplemental calcium 1.2 g per day. Regular exercise (30 used in this particular case.
minutes three times per week) and cessation of smoking
are also recommended. Since the Women’s Health Initiative study, which
demonstrated that HRT did not reduce the risk of
Drug treatment of choice in established osteoporosis cardiovascular disease and increased the risk of stroke,
is a bisphosphonate, of which alendronate has been used breast cancer and venous thromboembolism, oestrogen
most successfully. Other drugs include selective therapy is no longer first line and is only reserved for
oestrogen receptor modulators such as raloxifene, patients who do not tolerate other treatments. Calcitonin
oestrogen itself, calcitonin and vitamin D. The selective is effective but less practical to take because it is
receptor modulator raloxifene has been shown to be administered nasally and furthermore, calcitonin therapy
effective in the treatment of osteoporosis, and – unlike exhibits tachyphylaxis.

Answer 135

b. There is a high chance that the patient will be Cardiovascular disease is the commonest cause of
dialysis-dependent by the age of 60 years. death in PKD. Renal failure is the most serious renal
complication of PKD. The median age of end-stage renal
The features of polycystic kidney disease are covered in failure is 54 years; in patients with PKD1 and PKD2 it is
Question 382. Hypertension is common in patients with 74 years. There is a high probability that this patient has
PKD, and patients with hypertension have a faster decline PKD1 since PKD1 accounts for the vast majority of cases
in renal function than patients who are normotensive. of PKD and has an earlier presentation. The most serious
However, there is no conclusive trial demonstrating that extra-renal manifestation of PKD is subarachnoid
treatment of hypertension reduces the rate of decline of haemorrhage due to intracerebral aneurysms. Patients
renal function. ACE inhibitors and AIIRBs do not have with PKD should be screened for intracranial aneurysms
an effect on renal function but may improve prognosis by with magnetic resonance angiography if there is a family
preventing left ventricular hypertrophy. history of subarachnoid haemorrhage.

Answer 136

d. IV calcium gluconate. Management of hyperkalaemia

The patient has life-threatening hyperkalaemia, which is Method Agent
defined as a serum potassium level ≥7 mmol/l or
hyperkalaemia <7 mmol/l that is associated with typical Antagonism of Calcium gluconate
electrocardiographic changes or muscle weakness, the
latter occur owing to antagonistic effects of potassium on membrane actions
the membrane potential. In these circumstances the
immediate treatment is to stabilize the membrane of potassium
potential with intravenous calcium gluconate. Intravenous
calcium acts within minutes. Its effects last for up to 60 Drive extra-cellular Dextrose and insulin
minutes, which allows definitive treatment with potassium into cells Sodium bicarbonate if acidotic
intravenous dextrose and insulin to take effect (Table).
Beta-agonists

Removal of Diuretics

potassium from cells Cation exchange resins

Haemodialysis if severe or in

patients with renal failure

Clinical Cases 139

Question 137 Hb 10 g/dl

A 5-year-old male was admitted for investigation of ankle WCC 5 ϫ 109/l
swelling and abdominal distension that had occurred over
three months. His appetite was satisfactory, and there was Platelets 166 ϫ 109/l
no history of diarrhoea, breathlessness, arthralgia or
syncope. He was born by a normal vaginal delivery and MCV 80 fl
weighed 3.6 kg at birth. He was well as a neonate and had
achieved his milestones normally thus far. Apart from Urinalysis Microscopy revealed
having mild eczema there was no past medical history of
note. His father was a solicitor, and his mother had just occasional transitional cells
returned to full-time teaching after giving birth to his
16-month-old brother, who was the only other sibling. 24-hour urinary protein 5.2 g
There was no history of travel abroad or any drug history.
Selective protein clearance
On examination, he had facial swelling. There was no
pallor or cyanosis. The most striking feature was pitting ratio was low
oedema of the lower limbs from the ankles to the thighs.
The heart rate was 100 beats/min and regular. The blood Ascitic fluid Protein 8 g/l
pressure was 95/55 mmHg. The JVP was not raised.
Palpation of the precordium was normal. On auscultation, Chest X-ray Normal
heart sounds I and II were normal, but there was an
additional third heart sound. Examination of the Sodium 135 mmol/l
respiratory system revealed dullness to percussion at the
right lung base and reduced air entry on auscultation. Potassium 3.7 mmol/l
Abdominal examination demonstrated a non-tender,
distended abdomen with shifting dullness. There was no Urea 2.1 mmol/l
palpable organomegaly.
Creatinine 65 μmol/l
Investigations are shown.
The patient was treated with 60 mg prednisolone and Bilirubin 10 μmol/l
40 mg furosemide. The oedema started improving after
one week, and renal function remained normal. After two AST 19 iu/l
weeks he was discharged and seen at weekly intervals. In
the sixth week the oedema had completely subsided, and Alkaline phosphatase 80 iu/l
his abdomen was soft and non-distended. A 24-hour
urinary protein estimation was 0.7 g. The furosemide was Total protein 36 g/l
stopped and plans were made to review the patient again
in two weeks, but three days later he was admitted with Albumin 13 g/l
right upper quadrant pain and abdominal distension. On
examination, the JVP was not raised. There was palpable Glucose 4 mmol/l
tender hepatomegaly and shifting dullness. There was no
lower-limb oedema. Bilirubin 15 μmol/l
Further investigations are shown. AST 90 iu/l
Alkaline phosphatase 89 iu/l
Albumin 35 g/l

1. What was the original cause of oedema?
2. What was the subsequent cause of abdominal pain

and distension?
3. List two tests which will help confirm the cause of

abdominal pain.
4. In one step, what is the management?

Question 138 138

What is the rhythm disturbance in this rhythm strip
(138)?

140

Answer 137

1. Nephrotic syndrome. or chlorambucil. Complications of nephrotic syndrome
2. Hepatic vein thrombosis. include susceptibility to infection due to loss of
3. i. Ultrasound or CT scan of the liver looking immunoglobulins in the urine, protein malnutrition,
hyperlipidaemia, arterial and venous thromboses and
particularly at the hepatic and portal veins. acute renal failure.
ii. Liver biopsy.
4. Thrombolysis to prevent severe hepatic congestion. Thrombosis is a frequent and serious complication;
corticosteroids and diuretic therapy contribute to
The combination of hypoalbuminaemia, heavy thrombosis. Whole blood viscosity is increased, and there
proteinuria and oedema is strongly suggestive of the is alteration in the concentration of clotting factors in the
diagnosis of nephrotic syndrome, which in young blood. The levels of antithrombin III and plasminogen
children is most often due to minimal change are reduced, increasing the tendency to thrombosis. In
glomerulonephritis. Minimal change glomerulonephritis this case the onset of abdominal pain, tender right upper
is thought to be an auto-immune disorder, and usually quadrant, ascites and abnormal LFT reflect hepatic vein
responds extremely well to steroids and other or portal vein thrombosis. Patients may present rapidly
immunosuppressants such as cyclophosphamide and with fulminant hepatic failure or with gradual-onset
chlorambucil. There is a well-recognized association of ascites and jaundice. Early revascularization with a
the disorder with Hodgkin’s lymphoma, bee stings, cow’s thrombolytic agent is important to prevent hepatic
milk allergy and ingestion of NSAIDs. Minimal change infarction and fibrosis leading to chronic portal
nephropathy is found in 80% of children with nephrotic hypertension. In patients where thrombolysis is
syndrome. Treatment is with high-dose steroids, and contraindicated, a shunt procedure is necessary.
diuretics are often necessary to treat the oedema.
Relapses are common and are treated with steroids until The diagnosis can be confirmed by ultrasonography
the urine is completely free of protein for three days. and Doppler. The hepatic veins, portal vein and inferior
Steroids are prescribed for a maximum of four weeks. vena cava should be examined by venography. Liver
Recurrent relapses are managed with cyclophosphamide biopsy helps make the diagnosis, and is also important to
assess hepatic necrosis and fibrosis.

Answer 138

Atrial flutter with 2:1/3:1 atrioventricular block. Causes of atrial flutter/fibrillation

There is an irregular narrow complex tachycardia, and the • Ischaemic heart disease
differential diagnosis is between atrial fibrillation, atrial • Hypertension
flutter with varying atrioventricular block, and an atrial • Mitral stenosis
tachycardia with varying atrioventricular block. The clue • Thyrotoxicosis
is in the rhythm strip which reveals a ‘saw-tooth’ pattern • Myocarditis
to the ECG complexes, indicating flutter. The RR- • Cardiomyopathy
interval is not constant, therefore there is flutter with • Pericarditis
varying block. In this case there are between two and • Bronchial carcinoma
three flutter waves between each RR-interval, and hence • Severe pneumonia
the diagnosis is atrial flutter with 2:1/3:1 atrioventricular • Chronic lung disease
block. The causes of atrial flutter are the same as those • Pulmonary emboli
for AF. The most effective treatment for atrial flutter is • Wolff–Parkinson–White syndrome
DC cardioversion, delivering small energy shocks of • Alcohol abuse
between 50 and 100 J. Chemical cardioversion is • Cocaine abuse
difficult, but the most effective drugs for this purpose are
flecainide and amiodarone. Like AF, patients with atrial
flutter should be anticoagulated to prevent the risk of
thromboembolism.

Clinical Cases 141

Question 139 Hb 12 g/dl
WCC 5 ϫ 109/l
A 66-year-old retired school teacher presented with a six- Platelets 180 ϫ 109/l
month history of progressive weakness, fatigue and MCV 86 fl
breathlessness which was accompanied by weight loss of Sodium
5 kg. He had difficulty climbing stairs because his ‘legs Potassium 137 mmol/l
would not carry him’, and had noticed that he had Urea
difficulty holding light objects with his left hand without Creatinine 4.1 mmol/l
dropping them. Over the past week his wife had noticed Chest X-ray
that his speech appeared slurred and nasal, and he was 6 mmol/l
having difficulty swallowing his meals. He had a past 110 μmol/l
history of pernicious anaemia for which he was taking Normal
regular intramuscular B12 injections. On examination, he
was thin. There was no evidence of pallor or clubbing. 1. What is the diagnosis?
The patient had dysarthria. Examination of the cranial a. Cervical myelopathy.
nerves revealed normal eye movements, but tongue b. Subacute combined degeneration of the spinal
movement was sluggish and there was reduced palatal cord.
movement. The jaw jerk was brisk. On examination of his c. Multiple sclerosis.
limbs there was wasting and fasciculation of the small d. Motor neurone disease.
muscles of the left hand. The tone and power in the left e. Polymyositis.
upper limb was generally reduced. The upper-limb
reflexes were brisk. There was wasting and fasciculation in 2. What two investigations could be performed to
both thigh muscles. The tone was increased and the confirm your suspicion?
power was reduced. The ankle and knee jerks were brisk, a. Anti-Hu antibodies.
and there was obvious clonus at the ankle joint. Sensation b. MRI scan brain.
was normal and Romberg’s test was negative. Examina- c. CSF analysis for oligoclonal bands.
tion of the fundi revealed bitemporal pallor. d. Visual evoked potentials.
e. Nerve conduction studies.
Investigations are shown. f. MRI scan of cervical spine.
g. Serum creatinine kinase.
h. Serum B12 concentration.
i. Muscle biopsy.
j. Electromyography.

Question 140 140

An 86-year-old male was found collapsed in his home by 1. List four abnormalities on the ECG.
his neighbour after his son raised concern about not 2. Which single investigation should the medical
being able to contact him by telephone. There was no
significant past medical history. He was independent. He officer have performed?
was last seen two days earlier when he joined his 3. In one sentence, what is the management of this
neighbour for Christmas dinner.
case?
On examination, he was unconscious and had a
Glasgow coma scale of 5 out of 15. There was no nuchal
rigidity. Pupillary reflexes were sluggish, but examination
of the fundi was normal. Tone was slightly increased in
all the limbs. The peripheral reflexes were present and
both plantar reflexes were flexor. The heart rate was
40 beats/min, and blood pressure was 80/40 mmHg.
Heart sounds were normal, and the chest was clear.

An ECG was performed in the Accident and Emer-
gency Department (140).

142

Answer 139

1. d. Motor neurone disease. due to mutations in the copper/zinc superoxide dis-
2. e. Nerve conduction studies. mutase gene on chromosome 21. The incidence is
1/50,000, and the male-to-female ratio is 1.5:1. There
j. Electromyography. are three distinct patterns known as progressive muscular
atrophy, amyotrophic lateral sclerosis and progressive
There is evidence of combined lower motor neurone and bulbar palsy which are characterized by predominantly
upper motor neurone lesions and a pseudobulbar palsy. lower motor lesions, predominantly upper motor neurone
There is no sensory abnormality, and ocular movements are lesions, and bulbar and pseudobulbar palsy, respectively.
normal. These findings are characteristic of motor neurone Usually, a combination of all three is present. Female sex,
disease, which is characterized by a progressive bulbar onset and old age are poor prognostic markers.
degeneration of the motor neurones in the cortex, spinal The disease usually progresses inexorably and death is
cord and motor nuclei of the cranial nerves. The breath- usually from bronchopneumonia within five years of onset
lessness is probably due to involvement of the respiratory of symptoms.
muscles. The bitemporal pallor is a ‘red herring’, and al-
though it may have led some readers to diagnose multiple The diagnosis is clinical, but electromyography charac-
sclerosis, it is important to note that bitemporal pallor is a teristically reveals a reduced number of action potentials
normal finding in some individuals. Moreover, wasting and in the muscles which have an increased amplitude and
fasciculation are not features of multiple sclerosis because it duration. Nerve conduction studies reveal normal motor
does not affect lower motor neurones. Although the patient conduction and exclude an underlying neuropathy,
has a history of pernicious anaemia, his blood count does particularly in patients with the progressive muscular
not suggest B12 deficiency and therefore subacute atrophy variety of the disorder.
degeneration of the spinal cord is unlikely. The absence of a
sensory neuropathy is also against the diagnosis. There are no curative therapies, but glutamate antagon-
ists such as riluzole appear promising, as does ciliary
The cause of motor neurone disease is unknown. In neurotrophic factor, which has been shown to promote
approximately 15% of cases the disease is familial and is survival of cultured rat and human motor neurones. Both
agents are currently being tested on affected humans.

Answer 140

1. i. Bradycardia. hypopnoea, sluggish pupillary and peripheral reflexes,
ii. Tremor artefact. muscle rigidity and coma are recognized features of
iii. J-waves. severe hypothermia. Ventricular fibrillation and asystole
iv. Prolonged QT-interval. are the usual causes of death.

2. Rectal (core) temperature with a low-reading ECG abnormalities include bradycardia, atrial fibril-
thermometer. lation with a slow ventricular rate, prolonged PR- and
QT-intervals and J-waves. A baseline artefact due to
3. Gradual rewarming using space blankets and muscle tremor may also be seen, and should help make
possibly warmed intravenous fluids, depending on the diagnosis in the examination situation.
the severity of hypothermia.
Gentle rewarming is indicated. If the temperature is
Hypothermia is common among the elderly. It is usually above 32°C (89.6°F), space blankets and warm oral fluids
attributable to cold environment (no heating at home), are usually sufficient. If the temperature is lower than
inadequate clothing, poor nutrition, neuroleptic drugs, this, and the patient is unconscious, then warmed intra-
alcohol and hypothyroidism. Hypothermia is defined as venous fluids should be given. In this situation there is a
a fall in the core temperature to below 35°C (95°F). danger of severe metabolic acidosis due to sluggish
Severe hypothermia causes impaired consciousness and circulation, which should be corrected promptly. The aim
cardiac embarrassment. Bradycardia, hypotension, is to increase the temperature by 1°C (1.8°F) per hour.

Question 141 Clinical Cases 143

141a

A 21-year-old chef presented with a two-week history of Hb 11 g/dl
general malaise, cough and headaches. He had just returned
from a ten-day holiday in a hotel resort in Spain. He WCC 14 ϫ 109/l (neutrophil leucocytosis)
consulted his GP on arrival, who diagnosed a respiratory
tract infection and prescribed erythromycin. Despite this, Platelets 480 ϫ 109/l
the patient’s symptoms continued to persist. He had now
developed a foul-smelling, blood-stained nasal discharge ESR 110 mm/h
and was deaf in his left ear. His cough was worse, but was
non-productive. He felt ‘hot and cold’ and had a reduced CRP 209 g/l
appetite due to nausea. He was a non-smoker and
consumed 20 units of alcohol per week. Sodium 134 mmol/l

On examination, he appeared relatively well. There was Potassium 4.1 mmol/l
no skin rash, pallor, cyanosis, clubbing, lymphadenopathy
or ankle oedema. He was tender over his frontal sinuses, Urea 6 mmol/l
and had crusting around the nasal septum on the right side.
Inspection of the outer ears and tympanic membranes was Creatinine 89 μmol/l
normal. The Rinne’s test revealed reduced air and bone
conduction on the left side. Weber’s test lateralized to the Urinalysis Protein +1
right side. Apart from the abnormal hearing tests, all other
neurological examination was normal. The heart rate was Blood not detected
96 beats/min and blood pressure 140/90 mmHg. The
patient was tachypnoeic on mild exertion, and had a Chest X-ray (141a)
respiratory rate of 20/min. Chest expansion was moderate.
The trachea was central. On auscultation of the lung fields 1. Describe the abnormalities on the chest
there was evidence of bronchial breathing at the left lung X-ray.
base and a few inspiratory crackles at the right lung base.
Examination of the heart and abdomen was normal. 2. How would you interpret the findings
from the Rinne’s and Weber’s tests?
Investigations are shown.
3. What two tests could you perform to
help achieve the diagnosis?

4. What is the underlying diagnosis?

144

Answer 141

1. The chest X-ray demonstrates alveolar shadowing involvement. The kidneys do not appear to be significantly
affecting the right middle and lower lobes and in affected. The trace of protein may be a response to a febrile
the left lower lobe. In addition, there is a round illness. Upper respiratory involvement can also lead to
cavitating lesion in the left lower lobe. The findings hoarseness and stridor due to vocal cord and tracheal
are consistent with either a purulent pneumonia involvement, respectively, and lower respiratory
with left lower lobe abscess formation, or involvement may present with pleurisy and haemoptysis.
pulmonary haemorrhage and a cavitating Renal involvement can vary from asymptomatic proteinuria
granuloma due to Wegener’s granulomatosis. and haematuria to fulminant renal failure due to crescentic
glomerulonephritis. Other organs affected include the eye,
2. The Rinne’s test reveals reduced bone and air cardiovascular system, central nervous system and the skin.
hearing conduction, suggesting sensorineuronal In the eye, the disease can present with scleritis, uveitis and
deafness on the left side. The Weber’s test does not pseudotumour due to retro-orbital granuloma formation.
normally lateralize to either ear, but in this case it Cardiovascular involvement is rare. The most common
lateralized to the right. This is in keeping with manifestation is pericarditis, but conduction tissue
sensorineuronal hearing loss in the left ear. Because involvement produces tachy- and bradyarrhythmias.
both air and bone conduction are impaired in Involvement of the central nervous system is also rare and
sensorineuronal deafness, the sound from the comprises a mononeuritis multiplex, isolated cranial nerve
tuning fork (which is conducted via bone in the palsies and meningeal involvement.
Weber’s test) is heard loudest in the ear which is
not affected. In the case of pure conduction The diagnosis is based on demonstrating a necrotizing
deafness, the Weber’s test lateralizes to the affected granulomatous vasculitis biopsy of the affected tissue.
ear. Wegener’s granulomatosis can produce The best yield of a positive biopsy comes from the kidney
conduction and sensorineuronal deafness by or the lungs when they are involved. ANCA are present
blockage of the Eustachian tube when it involves in over 80% of untreated cases. Immunofluorescence
the upper respiratory tract and 8th cranial nerve, studies have revealed two main types of ANCA which are
respectively. determined by the pattern of cytoplasmic staining.
c-ANCA stains the outer cytoplasm, whereas p-ANCA
3. i. Nasal or transbronchial lung biopsy. stains the perinuclear cytoplasm. It is now clear that
ii. c-ANCA (antiproteinase 3). c-ANCA binds the enzyme proteinase 3 (PF3) and is
specific for Wegener’s granulomatosis. p-ANCA is
4. Wegener’s granulomatosis. heterogeneous. One subset binds the cytoplasmic enzyme
MPO, which is positive in microscopic polyangiitis and
Wegener’s granulomatosis is a necrotizing granulomatous idiopathic crescentic glomerulonephritis. Other subsets of
vasculitis affecting the upper and lower respiratory tracts p-ANCA bind elastase, lactoferrin, lysozyme and
and the kidneys, although several other organs can be cathepsin G and are present in a variety of auto-immune
affected. Granulomas occur in small arterioles. The conditions including CAH, sclerosing cholangitis,
diagnosis should be suspected when two of these systems ulcerative colitis and SLE (141b).
are involved in the presence of systemic features such as
malaise, night sweats, fever and weight loss. This patient The management of Wegener’s granulomatosis is with
has features of upper and lower respiratory tract steroids and other immunosuppressants such as
involvement which include frontal sinusitis, purulent nasal cyclophosphamide. Disease progress is monitored by
discharge and nasal septal crusting, dry cough and evidence estimating ANCA titres. There is good correlation
of gross respiratory involvement on the chest X-ray. The between ANCA titres and underlying inflammatory
deafness in the left ear is indicative of 8th nerve activity in most patients.

ANCA p-ANCA 141b

c-ANCA Anti-MPO subset Non-specific subset
(PF3)
Microscopic polyarteritis A variety of autoimmune diseases:
Wegener’s granulomatosis nodosa SLE

Idiopathic crescentic Mixed connective tissue disease
glomerulonephritis Ulcerative colitis
RTA
CFA

Clinical Cases 145

Question 142

A 28-year-old male presented with a three-day history of
rigors and pleuritic pain affecting the left lung, where
there were signs of consolidation. There was no evidence
of lymphadenopathy or hepatosplenomegaly.

Investigations are shown.

Hb 12 g/dl 1. What is the characteristic term given to this blood
WCC 24 ϫ 109/l picture?
Platelets 149 ϫ 109/l
Neutrophils 85% 2. Which two features on the blood film would
Normoblasts 7% confirm your suspicions?
Myeloblasts 3%
Myelocytes 2%
Lymphocytes 3%

Question 143

The following are serial LFT on a 51-year-old married He consumed approximately 4 units of alcohol per week.
schoolteacher who had a partial gastrectomy for a Apart from very slight jaundice, the physical examination
perforated peptic ulcer ten years previously. He was was normal.
asymptomatic. He was currently taking ranitidine.

Alanine aminotransferase (normal 5–30 iu/l) 1987 1988 1990 1992 1993
Aspartate aminotransferase (normal 10–40 iu/l) 30 36 40 44 49
Alkaline phosphatase (normal 25–115 iu/l) 42 65 69 73 78
Bilirubin (normal <17 μmol/l)
100 112 118 125 135
17 20 24 29 36

1. What is the most likely cause for the abnormality of
the LFT?

2. Which two investigations would you perform?
3. What treatment should be considered?

Question 144

The following are cardiac catheter data on a 4-year-old male with cyanosis and failure to thrive:

Chamber Pressure Oxygen saturation Oxygen saturation (%) 1. List four abnormalities.
(mmHg) (%) (on air) (on 28% oxygen) 2. What is the underlying
Right atrium
Right ventricle 8 52 52 diagnosis?
Pulmonary artery 90/30 53 54 3. Explain the lack of rise
PCWP 22/12 53 54
Left ventricle in oxygen tension after
Aorta 4 inspiring 28% oxygen.
80/20
77/50 82 81
66 66

146

Answer 142 A leukaemoid reaction is a descriptive term given to a
leucoerythroblastic blood picture which follows sepsis. In
1. ‘Leukaemoid reaction’. some cases of overwhelming infection, the ‘sick’ marrow
2. i. The presence of toxic granulations or Dohle releases immature cells into the blood stream; thus, there
is an abundance of myelocytes and normoblasts. In con-
bodies in the white cells. trast to a true leucoerythroblastic anaemia where the mar-
ii. Neutrophils exhibiting a toxic shift to the left, row is characteristically replaced by neoplastic cells, there
is no lymphadenopathy or hepatosplenomegaly.
i.e. three or fewer segments of the nucleus.

Answer 143

1. Chronic active hepatitis due to hepatitis B or severity of the hepatic damage. In hepatitis B-related CAH,
hepatitis C. the hepatitis B surface antigen and the e antigen is present.
In hepatitis C-related CAH, antihepatitis C antibodies are
2. i. Hepatitis B and C serology. present. The histological landmark of CAH is piecemeal
ii. Liver biopsy. necrosis. There is destruction of the liver cells between the
interface of the hepatic parenchyma and the connective
3. Interferon therapy. tissue. In hepatitis B-related CAH, the presence of hepatitis
B surface antigen in the hepatocytes produces a ground-
The patient has had major surgery in the past that almost glass appearance on haematoxylin and eosin staining.
certainly required blood transfusion. Both hepatitis B and C
are readily contracted from blood transfusions. Since Patients with evidence of CAH should be treated with
screening donated blood for hepatitis B was initiated in the intravenous interferon for several months to prevent viral
1980s, hepatitis C has become the most common cause of replication and allow seroconversion. The success rate is
post-transfusion hepatitis in the Western world. below 30%, and relapse often occurs soon after cessation of
therapy.
Almost 20% of patients with hepatitis B infection will
develop CAH. The cause is unknown, but is thought to be Other causes of CAH
a defective T-cell immune response. Most patients affected
are male. The condition is usually asymptomatic, or may Autoimmune
present as very slowly progressive hepatitis. About 50% of • Lupoid hepatitis
those affected present with established chronic liver disease. • Anti-LKM antibody-associated CAH

CAH may also occur following hepatitis C infection. Drugs
The acute hepatitic event is usually very mild, and often • Methyl dopa
goes unnoticed. The sequelae are similar to those of hepa- • Isoniazid
titis B infection; however the incidence of CAH is almost • α-1 antitrypsin deficiency
50%. Other causes of CAH are given (Table). There is • Wilson’s disease
nothing in the patient’s history to suggest any of these • Inflammatory bowel disease (usually ulcerative
other causes.
colitis)
The aminotransferases and bilirubin are usually modestly
raised and the alkaline phosphatase is very slightly raised.
The blood transaminase level bears no correlation with the

Answer 144

(See Interpretation of Cardiac Catheter Data, page 418.)

1. i. Elevated right ventricular pressure. 2. Fallot’s tetralogy.
ii. Pressure drop across the pulmonary valve, 3. The right-to-left shunt means that the majority of
suggesting pulmonary stenosis.
iii. Low oxygen saturation in the left ventricle, the blood does not pass through the lungs to be
suggestive of a VSD with a right-to-left shunt. oxygenated.
iv. Saturation in the ascending aorta much lower
than the left ventricle, indicating an overriding
aorta.

Clinical Cases 147

Question 145

145a 145b

Hearing level (dB) -10 AC
0 Right
Left
10
20 BC
30 Right
40 Left
50 Unmasked
60
70 500 1000 2000 4000 8000 Frequency (Hz)
80
90 500 1000 2000 4000 8000 Frequency (Hz)
100
110
120
130
140

125 250

A 30-year-old male presented with vertigo and deafness Hearing level (dB) -10
in the left ear. His trunk was examined (145a), and he 0
had an audiogram as part of the investigation for his 250
symptoms (145b). There was no air–bone gap. 10
20
1. What is the audiographic abnormality? 30
2. What is the unifying diagnosis? 40
50
3. What other abnormalities would you expect to find 60
on neurological examination? 70
80
90
100
110
120
130
140

125

Question 146

A 17-year-old male was referred with poorly developed On examination, there was absence of axillary and pubic
secondary sexual characteristics. He was a slow developer, hair. His penis and testicles were small. There was bilateral
and had been to a special school for his entire education. gynaecomastia. Neurological examination was normal.
He had a brother who was severely handicapped and
wheelchair-bound who had died at the age of 17 years. Investigations are shown.

Hb 12 g/dl 1. What is the endocrine abnormality?
2. What is the most probable diagnosis?
WCC 5 ϫ 109/l 3. How would you confirm the diagnosis?
4. How would you explain the abnormal creatinine
Platelets 190 ϫ 109/l
kinase level?
Sodium 137 mmol/l

Potassium 3.7 mmol/l

Urea 5 mmol/l

Creatinine kinase 350 iu/l

Testosterone 4 nmol/l

LH 35 iu/l (normal range 1–10 iu/l)

FSH 30 iu/l (normal range 1–7 iu/l)

148

Answer 145 145c

1. There is severe hearing loss at progressively higher
frequencies in the left ear.

2. The trunk demonstrates café au lait spots, a feature
of neurofibromatosis. An important association of
neurofibromatosis is acoustic neuroma, which is a
recognized cause of sensorineuronal deafness
(Table A).

3. On neurological examination, the patient may also
have an absent ipsilateral corneal reflex and
ipsilateral cerebellar signs. In a minority of cases
acoustic neuromas may be bilateral.

Neurofibromatosis is inherited as an autosomal dominant mandatory to detect acoustic neuroma early. MRI of the
condition. Abnormalities on two chromosomes may brain revealed an acoustic neuroma (145c).
cause disease with either a predominantly peripheral or
central manifestation (Table B). Regular audiometry is (See Audiograms, page 416.)

Table A Causes of sensorineuronal deafness Table B Other associations of neurofibromatosis

Unilateral (lesion in the cerebellopontine angle) Chromosome 17 (peripheral abnormalities)
• Meningioma • Renal artery stenosis
• Acoustic neuroma • Pulmonary fibrosis
• Granuloma • Cardiomyopathy (hypertrophic, restrictive and
• Metastasis
dilated)
Bilateral • Fibrous dysplasia of bone
• Degenerative (presbycusis) • Phaeochromocytoma
• Aminoglycosides
• Amphotericin Chromosome 22 (central abnormalities)
• High-dose loop diuretics • Acoustic neuroma
• Mumps • Meningioma
• Rubella • Optic glioma
• Ependymoma

Answer 146

1. Primary hypogonadism (hypergonadotrophic Leydig cell agenesis (Castelli’s syndrome), 5α-reductase
hypogonadism). deficiency, mumps orchitis, renal failure, sickle cell
disease, alcohol excess, chemotherapy and radiotherapy
2. Kleinfelter’s syndrome. (Table, Answer 260).
3. Demonstration of Barr body on buccal smear or
Patients have XXY karyotype due to non-dysjunction
karyotyping. and have the potential of being carriers for X-linked
4. Carrier for Duchenne muscular dystrophy. recessive conditions (as with females) such as Duchenne
muscular dystrophy, glucose-6-phosphate dehydrogenase
The most common cause of primary hypogonadism is deficiency and haemophilia A. Kleinfelter patients are
Kleinfelter’s syndrome, which has an incidence of infertile. They are capable of having erections and
1/1,000 births. Patients are tall, lack secondary sexual ejaculation, but the semen does not contain spermatozoa.
characteristics, have gynaecomastia, and are often men-
tally subnormal. The condition is characterized by failure Kleinfelter’s syndrome is associated with a higher
of development of Leydig cells and seminiferous tubules. incidence of carcinoma of the breast, hypothyroidism,
Other causes of primary hypogonadism include Prader– diabetes mellitus and chronic respiratory disease than the
Willi syndrome, Lawrence–Moon–Biedel syndrome, general male population.


Click to View FlipBook Version